You are on page 1of 70

CURRENT ELECTRICITY

THEORY AND EXERCISE BOOKLET

CONTENTS

S.NO. TOPIC PAGE NO.

1. Introduction ....................................................................................................... 3

2. Electric current and current density ................................................................ 3

3. Drift Current .................................................................................................... 4 – 6

4. Ohm's law ....................................................................................................... 6 – 9

5. Battery and EMF .......................................................................................... 10 – 12

6. Kirchhoff's Laws ........................................................................................... 12 – 17

7. Combination of Resistance ......................................................................... 17 – 22

8. Wheaststone's Bridge .................................................................................. 22 – 29

9. Combination of Cells ................................................................................... 29 – 32

10. Electrical Power ......................................................................................... 33 – 36

11. Instruments ................................................................................................. 36 – 45

12. Exercise - I .................................................................................................. 46 – 57

13. Exercise - II ................................................................................................. 58 – 59

14. Exercise - III ................................................................................................ 60 – 62

15. Exercise - IV ................................................................................................ 63 – 64

16. Exercise - V ................................................................................................ 65 – 68

17. Answer key ................................................................................................. 69 – 70

394,50 - Rajeev Gandhi Nagar Kota, Ph. No. : 93141-87482, 0744-2209671


IVRS No : 0744-2439051, 52, 53, www. motioniitjee.com , info@motioniitjee.com

Downloaded from www.iitjeephysics4u.com


Page # 2 CURRENT ELECTRICITY

Syllabus
Ohm's law; Series and parallel arrangements of resistances and
cells; kirchhoff's laws and simple applications.

394,50 - Rajeev Gandhi Nagar Kota, Ph. No. : 93141-87482, 0744-2209671


IVRS No : 0744-2439051, 52, 53, www. motioniitjee.com , info@motioniitjee.com
Downloaded from www.iitjeephysics4u.com
CURRENT ELECTRICITY Page # 3

1. INTRODUCTION
• Conductor
In some materials, the outer electrons of each atoms or molecules are only weakly bound to it. These
electrons are almost free to move throughout the body of the material and are called free electrons.
They are also known as conduction electrons. When such a material is placed in an electric field, the
free electrons move in a direction opposite to the field. Such materials are called conductors.
• Insulator :
Another class of materials is called insulators in which all the electrons are tightly bound to their
respective atoms or molecules. Effectively, there are no free electrons. When such a material is placed
in an electric field, the electrons may slightly shift opposite to the field but they can't leave their
parent atoms or molecules and hence can't move through long distances. Such materials are also
called dielectrics.
• Semiconductor :
In semiconductors, the behaviour is like an insulator at low levels of temperature. But at higher
temperatures, a small number of electrons are able to free themselves and they respond to the applied
electric field. As the number of free electrons in a semiconductor is much smaller then that in a
conductor, its behaviour is in between a conductor and an insulator and hence, the name semiconductor.
A freed electron in a semiconductor leaves a vacancy in its normal bound position. These vacancies
also help in conduction.
2. ELECTRIC CURRENT AND CURRENT DENSITY
When there is a transfer of charge from one side of an area to the other, we say that there is an
electric current through the area. If the moving charges are positive, the current is in the direction of
motion, if they are negative, the current is opposite to the direction of motion. If a charge Q crosses
an area in time t, we define the average electric current through the area during this time as
Q
i
t
The current at time t is
Q dQ
i  lim 
t  0 t dt
Thus, electric current through an area is the rate of transfer of charge from one side of the area to the
other. The SI unit of current is ampere. If one coulomb of charge crosses an area in one second, the
current is one ampere. It is one of the seven base units accepted in SI.

Ex.1 If q = 2t2 + 3, find current at t = 2 sec ?

dq
Sol. i
dt
i = 4t
 i at 2 sec = 4 × 2 = 8 A
We shall now define a vector quantity known as electric current density at a point. To define the
current density at a point P, we draw a small area S through P perpendicular to the flow of
charges(shown in figure) If i be the current through the area S, the average current density is
 i
j 
S
The current density at the point P is

i di
j  lim 
S  0 S dS
The direction of the current density is the same as the direction of the current. Thus, it is along the
motion of the moving charges, if the charges are positive and opposite to the motion of the charges, if

394,50 - Rajeev Gandhi Nagar Kota, Ph. No. : 93141-87482, 0744-2209671


IVRS No : 0744-2439051, 52, 53, www. motioniitjee.com , info@motioniitjee.com

Downloaded from www.iitjeephysics4u.com


Page # 4 CURRENT ELECTRICITY

the charges are negative. If a current i is uniformly distributed over an area S and is perpendicular to
it,

i
j
S

Q
P S cos 

j

Q S
 i
t S n̂
(a)
Now let us consider an area S which is not necessarily perpendicular to the current (figure shown) If
the normal to the area makes an angle  with the direction of the current, the current density is,

i
j
S cos 
or, i = j S cos 

where i is the current through S, If S be the area vector corresponding to the area S, we have
 
i  j.S
For a finite area,
 
i  j.dS
Note carefully that an electric current has direction as well as magnitude but it is not a vector
quantity. It does not add like vectors. Therefore current is neither a vector quantity nor a scalar
quantity but a tensor quantity. The current density is a vector quantity.

Ex.2 An electron beam has an aperature 1.0 mm2. A total of 6.0 × 1010 electrons go through any
perpendicular cross-section per second. Find (a) the current and (b) the current density in the
beam.
Sol. The total charge crossing a perpendicular cross-section in one second is
q = ne
= 6.0 × 1016 × 1.6 × 10–19 C
= 9.6 × 10–3 C
The current is

q 9.6  10 –3 C
i = = 9.6 × 10–3 A
t 1s
As the charge is negative, the current is opposite to be direction of motion of the beam.
(b) The current density is

i 9.6  10 –3 A 9.6  10 –3 A
j    9.6  10 3 A / m 2
S (1.0mm ) 2 1.0  10 – 6 m 2
3. DRIFT SPEED
A conductor contains a large number of loosely bound electrons which we call free electrons or
conduction electrons. The remaining material is a collection of relatively heavy positive ions which we
call lattice. These ions keep on vibrating about their mean positions. The average amplitude depends

394,50 - Rajeev Gandhi Nagar Kota, Ph. No. : 93141-87482, 0744-2209671


IVRS No : 0744-2439051, 52, 53, www. motioniitjee.com , info@motioniitjee.com
Downloaded from www.iitjeephysics4u.com
CURRENT ELECTRICITY Page # 5

on the temperature. Occasionally, a free electron collides or interacts in some other fashion with the
lattice. The speed and direction of the electron changes randomly at each such event. As a result, the
electron moves in a zig-zag path. As there is a large number of free electrons moving in random
directions, the number of electrons crossing an area S from one side very nearly equals the number
crossing from the other side in any given time interval. The electric current through the area is,
therefore, zero.
When there is an electric field inside the conductor, a force acts on each electron in the direction
opposite to the field. The electrons get biased in their random motion in favour of the force. As a
result, the electrons drift slowly in this direction. At each collision, the electron starts afresh in a
random direction with a random speed but gains an additional velocity v' due to the electric field. This
velocity v' increases with time and suddenly becomes zero as the electron makes a collision with the
lattice and starts afresh with a random velocity. As. the time ,t between successive collisions is small,
the electron "slowly and steadily drifts opposite to the applied field (shown figure). If the electron
drifts a distance  in a long time t, we define drift speed as
l
vd 
t
If  be the average time between successive collisions, the distance drifted during this period is
1 1  eE  2
l (  ) 2   ( )
2 2 m 
l 1  eE 
The drift speed is vd    
 2 m 
It is proportional to the electric field E and to the average collision-time .
The random motion of free electrons does not contribute to the drift of these electrons. Also, the
average collision-time is constant for a given material at a given temperature. We, therefore, make the
following assumption for our present purpose of discussing electric current.
When no electric field exists in a conductor, the free electrons stay at rest (Vd = 0) and when a field E
exists, they move with a constant velocity
e
vd  E  kE ...(1)
2m
opposite to the field. The constant k depends on the material of the conductor and its temperature.
E
A vd

v d t
Let us now find the relation between the current density and the drift speed. Consider a cylindrical
conductor of cross-sectional area A in which an electric field E exists. Consider a length vd t of the
conductor (figure shown). The volume of this portion is Avdt. If there are n free electrons per unit
volume of the wire, the number of free electrons in this portion is nAvdt. All these electrons cross the
area A in time t. Thus, the charge crossing this area in time t is
Q = nAvd t e
Q
or, i = nAvde
t
i
and j=  nev d ...(2)
A
Ex.3 Calculate the drift speed of the electrons when 1 A of current exists in a copper wire of cross-
section 2 mm2. The number of free electrons in 1 cm3 of copper is 8.5 × 1022.
Sol. We have
j = nevd

j i 1A
or, vd   =  0.036 mm / s
ne A ne (2  10 m )(8.5  10  10 6 m – 3 )(1.6  10 –19 C )
–6 2 22

We see that the drift speed is indeed small.

394,50 - Rajeev Gandhi Nagar Kota, Ph. No. : 93141-87482, 0744-2209671


IVRS No : 0744-2439051, 52, 53, www. motioniitjee.com , info@motioniitjee.com

Downloaded from www.iitjeephysics4u.com


Page # 6 CURRENT ELECTRICITY

4. OHM'S LAW
Using equations (1) and (2)

ne 2 
j  nev d  E
2m

or, j  E ...(3)

ne 2 
where  =
2m
where  depends only on material of the conductor and its temperature. This constant is called the
electrical conductivity of the material. Equation (3) is known as Ohm's law.
The resistivity of a material is defined as
1 2m
 = ...(4)
 ne 2 
Ohm's law tells us that the conductivity (or resistivity) of a material is independent of the electric field
existing in the material. This is valid for conductors over a wide range of field.
Suppose we have a conductor of length l and uniform cross-sectional area A (figure shown) Let us apply
a potential difference V between the ends of the conductor. The electric field inside the conductor is
V i
E . If the current in the conductor is i, the current density is j  . Ohm's law j = E then becomes
l A
v

i
A
E
l
(a) (b)
i V

A l
1 l l
or, V i i ...(5)
A A
or, V=Ri ...(6)
R is called the resistance of the given conductor. The quantity 1/ R is called conductance.
Equation (5) is another form of Ohm's law which is widely used in circuit analysis. The unit of resistance
is called ohm and is denoted by symbol . An object of conducting material, having a resistance of
desired value, is called a resistor.
From equation (5) and (6)


R ...(7)
A

From equation (7), the unit of resistivity  is ohm-metre, also written as -m. The unit of conductivity

() is (ohm-m) written as mho/m.

from eq. (4) & (7)


2 m 
R 2

ne A A
 2 m 
where  = resistivity  where   ne 2  

 
 = length along the direction of current

394,50 - Rajeev Gandhi Nagar Kota, Ph. No. : 93141-87482, 0744-2209671


IVRS No : 0744-2439051, 52, 53, www. motioniitjee.com , info@motioniitjee.com
Downloaded from www.iitjeephysics4u.com
CURRENT ELECTRICITY Page # 7

A = Area of the cross section perpendicular to direction of current

n = no. of free charges per unit volume.

 = relaxation time

m = mass of electron

Ex.4 Calculate the resistance of an aluminium wire of length 50 cm and cross-sectional area 2.0

mm2. The resistivity of aluminium is  = 2.6 × 10–8 -m



Sol. The resistance is R = 
A
( 2.6  10 –8   m)  (0.50 m)
  0.0065 
2  10 – 6 m 2
We arrived at Ohm's law by making several assumptions about the existence and behaviour of the free
electrons. These assumption are not valid for semiconductors, insulators, solutions etc. Ohm's law

cannot be applied in such cases.

Ex.5 The dimensions of a conductor of specific resistance  are shown below. Find the resistance of

the conductor across AB, CD and EF.

A D

c F
E
b
a
C
B

c b a
Sol. R AB  , R CD  , R EF 
ab ac bc

Ex.6 A portion of length L is cut out of a conical solid wire. The two ends of this portion have circular
cross-sections of radii r1 and r2 (r2 > r1). It is connected lengthwise to a circuit and a current i
is flowing in it. The resistivity of the material of the wire is . Calculate the resistance of the
considered portion and the voltage developed across it.
Sol. If follows from the figure, that

r2 – r1
tan = i r2
L r1

 r2 – r1  l
r L  x(r2 – r1 )
 r = r1 + x tan  = r1 + x   = 1
 L  L

394,50 - Rajeev Gandhi Nagar Kota, Ph. No. : 93141-87482, 0744-2209671


IVRS No : 0744-2439051, 52, 53, www. motioniitjee.com , info@motioniitjee.com

Downloaded from www.iitjeephysics4u.com


Page # 8 CURRENT ELECTRICITY

2 
 A  r  [r1L  (r2 – r1 )x ]2
L2

dx dxL2 L2


L
dx
dR    R  dR 
r 2 [r1L  (r2 – r1 ) x ] 2    [r L  (r – r1 )x ] 2
0 1 2

L2 L I 
 {r1L  (r2 – r1 )x } –1 0  –
  
  (r2 – r1 ) 

– L  1 1  L IL
  – 
(r2 – r1 )  r2 r1  (r1r2 )  V  IR  r r
12

Ex.7 The space between two coaxial cylinders, whose radii are a and b (where a < b as in (figure
shown) is filled with a conducting medium. The specific conductivity of the medium is .

a
b

(a) Compute the resistance along the length of cylinder.


(b) Compute the resistance between the cylinders in the radial
direction. Assume that the cylinders are very long as compared to
their radii, i.e., L >> b, where L is the length of the cylinders.

l l l l
Sol. (a) R = = A  2 2
 2
A ( b – a ) (b – a2 )
(b) From Ohm's law, we have
 
J  E

Assuming radial current density. J becomes

 I
J r̂ for a < r < b
2rL
 I
and, therefore, E r̂
2rL
 
Here we have used the assumption that L >> b so that E and J are in cylindrically symmetric form.
The potential drop across the medium is thus :
a a
  I dr I b
Vab   E(r ). dr = –
 r  ln  
2L 2L  a 
b b
The resistance
b
In 
Vab a
R ab    
I 2L

394,50 - Rajeev Gandhi Nagar Kota, Ph. No. : 93141-87482, 0744-2209671


IVRS No : 0744-2439051, 52, 53, www. motioniitjee.com , info@motioniitjee.com
Downloaded from www.iitjeephysics4u.com
CURRENT ELECTRICITY Page # 9

Method 2 : We split the medium into differential cylindrical shell elements of width dr, in series. The current
flow is cylindrically symmetric (L >> b). The area through which the current flows across a shell of
radius r is A(r) = 2rL. The length the current flows, passing through a shell of radius r is dr. Therefore,
the resistance of the shell of radius r is :

1 dr
dR 
 2rL
Since the shells are connected in a series, we have

b
b ln 
a
R ab  dR   
 2L
a

Effect of Temperature on Resistance


(a) Resistance of Pure Metals

 2m  l
(i) We know that R 2 
 ne   A
For a given conductor, l, A and n are constant, hence R  (1/)
If  represents the mean free path (Average distance covered between two successive collisions) of
the electron and vrms, the root-mean-square speed, then
 Vrms
 , Hence R 
v rms 
Now,
(a)  decreases with rise in temperature because the amplitude of vibrations of the +ve ions of the metal
increases and they create more hindrance in the movement of electrons and,
(b) (i) vrms increases because vrms   T. Therefore, Resistance of the metallic wire increases with rise
in temperature. As  R and   (1/), hence resistivity increases and conductivity decreases with
rise in temperature of the metallic of the metallic wires.
(ii) If R0 and Rt represent the resistances of metallic wire at 0°C and t°C respectively then Rt is given by
the following formula :
Rt = R0(1 +  t)
where  is called as the Temperature coefficient of resistance of the material of the wire.
 depends on material and temperature but generally it is taken as a constant for a particular material
for small change.
Rt – R0 = R0  t
for very small change in temperature dR = R0  dt

(c) Resistance of semiconductors


(i) There are certain substances whose conductivity lies in between that of insulators and conductors,
higher than that of insulators but lower than that of conductors. These are called as semiconductors,
e.g., silicon, germanium, carbon etc.
(ii) The resistivity of semiconductors decreases with increase in temperature i.e.,  for semiconduc-
tors is –ve and high.
(iii) Though at ordinary temperature the value of n (no. of free electrons per unit volume) for these
materials is very small as compared to metals, but increases very rapidly with rise in temperature (this
happens due to breaking of covalent bonds). Though  decreases but factor of n dominates. There-
fore, the resistance
ml
R goes on decreasing with increase in temperature.
ne2 A

394,50 - Rajeev Gandhi Nagar Kota, Ph. No. : 93141-87482, 0744-2209671


IVRS No : 0744-2439051, 52, 53, www. motioniitjee.com , info@motioniitjee.com

Downloaded from www.iitjeephysics4u.com


Page # 10 CURRENT ELECTRICITY

5. BATTERY AND EMF


A battery is a device which maintains a potential difference between its two terminals A and B. Figure
shows a schematic diagram of a battery. Some internal mechanism exerts forces on the charges of the
battery material. This force drives the positive charges of the battery material towards A and the

negative charges of the battery material towards B. We show the force on a positive charge q as Fb .
As positive charge accumulates on A and negative charge on B, a potential difference develops and

grows between A and B. An electric field E is developed in the battery material from A to B and exerts
  
a force Fe  qE on a charge q. The direction of this force is opposite to that of Fb In steady state, the
charge accumulation on A and B is such that Fb  Fe . No further accumulation takes place.

+ –
Fb B
A

If a charge q is taken from the terminal B to the terminal A , the work done by the battery force Fb is
W = Fb d where d is the distance between A and B. The work done by the battery force per unit charge
is

W Fb d
E 
q q
This quantity is called the emf of the battery. The full form of emf is electromotive force. The name is
misleading in the sense that emf is not a force, it is work done/charge. We shall continue to denote this
quantity by the short name emf. If nothing is connected externally between A and B,
Fb = Fe = qE
or, Fbd = qEd = qV
where V = Ed is the potential difference between the terminals. Thus,

Fb d
E V
q
Thus, the emf of a battery equals the potential difference between its terminals when the terminals
are not connected externally.
Potential difference and emf are two different quantities whose magnitudes may be equal in certain
conditions. The emf is the work done per unit charge by the battery force Fb which is non-electrostatic
in nature. The potential difference originates from the electrostatic field created by the charges
accumulated on the terminals of the battery.
A battery is often prepared by putting two rods or plates of different metals in a chemical solution.
Such a battery, using chemical reactions to generate emf, is often called a cell.

E

A B

Now suppose the terminals of a battery are connected by a conducting wire as shown in above figure.
As the terminal A is at a higher potential than B, there is an electric field in the wire in the direction
shown in the figure. The free electrons in the wire move in the opposite direction .and enter the
battery at the terminal A. Some electrons are withdrawn from the terminal B which enter the wire
through the right end. Thus, the potential difference between A and B tends to decrease. If this
potential difference decreases, the electrostatic force Fe inside the battery also decreases. The force

394,50 - Rajeev Gandhi Nagar Kota, Ph. No. : 93141-87482, 0744-2209671


IVRS No : 0744-2439051, 52, 53, www. motioniitjee.com , info@motioniitjee.com
Downloaded from www.iitjeephysics4u.com
CURRENT ELECTRICITY Page # 11

Fb due to the battery mechanism remains the same. Thus, there is a net force on the positive charges
of the battery material from B to A. The positive charges rush towards A and neutralise the effect of
the electrons coming at A from the wire. Similarly, the negative charges rush towards B. Thus, the
potential difference between A and B is maintained.
For calculation of current, motion of a positive charge in one direction is equivalent to the motion of a
negative charge in opposite direction. Using this fact, We can describe the above situation by a simpler
model. The positive terminal of the battery supplies positive charges to the wire. These charges are
pushed through the wire by the electric field and they reach the negative terminal of the battery. The
battery mechanism drives these charges back to the positive terminal against the electric fIeld existing
in the battery and the process continues. This maintains a steady current in the circuit
Current can also be driven into a battery in the reverse direction. In such a case, positive charge enters
the battery at the positive terminal, moves inside the battery to the· negative terminal and leaves the
battery from the negative terminal. Such a process is called charging of the battery. The more common
process in which. the positive charge comes out of the battery from the positive terminal is called
discharging of the battery.

i =2A
A B
Ex.8 vA R  2 vB
E = 10 V
Find vA – vB
Sol. vA – iR – E = vB
vA – vB = iR + E = 4 + 10 = 14 volt

Ex.9 Shown in the figure. Find out the current in the wire BD

5
C B
20v 10v

D A
2

Sol. Let at point D potential = 0 and write the potential of

10
other points then current in wire AD = = 5A from A to D
2
v=+20 5 v=0
20 C B
current in wire CB = = 4A from C to B 4A
5 5A
4A
 current in wire BD = 1 A from D to B 10v

20v 1A + 10v
Ex.10 Find the current in each wire

H G F E v=0 A
D 5A 2
10v 30v 40v
50v
1 2
2
A B C D

394,50 - Rajeev Gandhi Nagar Kota, Ph. No. : 93141-87482, 0744-2209671


IVRS No : 0744-2439051, 52, 53, www. motioniitjee.com , info@motioniitjee.com

Downloaded from www.iitjeephysics4u.com


Page # 12 CURRENT ELECTRICITY

Sol. Let potential at point A is 0 volt then potential of other points is shown in figure.

40 – 0 +10V
Current in BG = = 40 A from G to B G10V 35A F
H
1 E
10V 5A 10V 20A
30V 50V
0 – (–30 ) 40V
Current in FC = = 15A from C to K 40V –30V –40V
2
10V
1 2 2
0 – (–40) 5A
current in DE =  20 A from D to E 40A 15A 20A
2
0V 0V
current in wire AH = 40 – 35 = 5 A from A to H A B C 20V D
0V 0V

6. KIRCHHOFF'S LAWS FOR CIRCUIT ANALYSIS


Before moving on to the statement of Kirchhoff's law, we state some conventions to be followed in
circuit analysis :
(1) Direction of conventional current is from high potential to low potential terminal.
(2) Current flows from high potential node A to low potential node B. if we traverse from point A to B,
there is drop of potential; similarly from B to A, there is gain of potential.
If we traverse from point A to B, there is drop of potential; similarly from B to A, there is gain of
potential. If a source of emf is traversed from negative to positive terminal, the change in potential is
+E.
Potential gain
VA – VB = + E
High Potential Low potential
A B
Final Initial
Potential drop potential potential

While discharging, current is drawn from the battery, the current comes out from positive terminal
and enters negative terminal, while charging of battery current is forced from positive terminal of the
battery to negative terminal. Irrespective of direction of current through a battery the sign conven-
tion mentioned above holds.
The positive plate of a capacitor is at high potential and negative plate at low potential. If we
traverse a capacitor from positive plate to negative plate, the change in potential is –Q/C
i + –
VB – VA = –Q/C
A B
Discharging Battery

i + – High Low
A B potential potential
Charging Battery

If we traverse a resistor in the direction of current, the change in potential is –IR.


High Potential Low Potential
+ –
A B VB – VA = –IR

Direction of traverse
Final Initial
Drop of potential
potential potential
Direction of traverse
Gain of potential
If we traverse a resistor in the direction opposite to the direction of current, the change in potential
is +IR.

394,50 - Rajeev Gandhi Nagar Kota, Ph. No. : 93141-87482, 0744-2209671


IVRS No : 0744-2439051, 52, 53, www. motioniitjee.com , info@motioniitjee.com
Downloaded from www.iitjeephysics4u.com
CURRENT ELECTRICITY Page # 13

Potential gain

Potential drop VA – VB = +IR

High Potential Low Potential Final Initial


+ – potential potential
VA VB

Positive terminal of source of emf is at high potential and negative terminal at low potential.
If we traverse a source of emf from the positive terminal to negative terminal, the change in
potential is –E.

Direction of traverse
VB – VA =–E
VB – VA = – E

Direction of traverse Final Initial


potential potential
VA – VB = + E

If a capacitor is traversed from negative plate to positive plate, the change in potential is
+Q/C.

VA – VB = + Q/C + –

High Low
potential potential
A B

(a) The Kirchhoff's Current Law


The Kirchhoff's current law (KCL) states that the algebraic sum
of the currents entering the junction must equal to sum of the I1 I2
O
currents leaving the junction. From the standard point of phys- I3
ics, KCL is a statement of charge conservation. I6
I4
The KCL applied to junction O yields.
I1 + I5 + I3 = I2 + I4

Outgoing Incoming
current current

2A

5A
3A

394,50 - Rajeev Gandhi Nagar Kota, Ph. No. : 93141-87482, 0744-2209671


IVRS No : 0744-2439051, 52, 53, www. motioniitjee.com , info@motioniitjee.com

Downloaded from www.iitjeephysics4u.com


Page # 14 CURRENT ELECTRICITY

20V

2

15V
1 2
–50 V
+10 V xA
Ex.11 Find the potential at point A 20V 5V

1

–30V
Sol. Let potential at A = x, applying kirchhoff current law at junction A

x – 20 – 10 x – 15 – 20 x – 5  50 x  30
   0
1 2 2 1

2x – 60  x – 35  x  45  2x  60
 0
2
 6x + 10 = 0

5
 x–
3

–5
Potential at A = V
3

2 2 2

Ex.12
5V 10V 20V

Find the current in every branch ?


Sol. Let we assume x potential at the top junction & zero potential at lower junction
i1 x i3
As from KCL,net current on a junction is O
i1 + i2 + i3 = 0 i2

x – 5 x – 10 x – 20
  0 2 2 2
2 2 2

35 10V
3x = 35  x  5V 20V
3
0 0 0
35
–5
10 A
 i1  3 
2 3

5 25
Similarly, i2 = A ; i3 = – A.
6 6

394,50 - Rajeev Gandhi Nagar Kota, Ph. No. : 93141-87482, 0744-2209671


IVRS No : 0744-2439051, 52, 53, www. motioniitjee.com , info@motioniitjee.com
Downloaded from www.iitjeephysics4u.com
CURRENT ELECTRICITY Page # 15

2

4V 4
2V 4V
2
Ex.13 2V
4

Find the current in every branch ?


Sol. Assume x potential at the upper junction & zero potential at the lower junction.
By KCl, we know that net current on a junction is zero. x i4 2
 i1 + i2 + i3 + i4 = 0
i3
i2
x–4 x–2 x2 x–4 4V
i1
   0 4
4V
2 4 4 2 2V
2x – 8 + x – 2 + x + 2 + 2x – 8 = 0 2
2V
4
8
6x – 16 = 0  x V
3
O O O O
2 1 7 2
 i1  – A, i2   A, i3  A, i 4  – A
3 6 6 3
2

2 2 2 2

Ex.14
4V 2V 2V

Find the current in every branch ?


Sol. The above question could be solved by assuming potential x & y at the top junctions & zero potential at
lower junctions
At the junction 1 applying KCL, i1 x i3 2 i4 y i6
i1+ i2 + i3 = 0 1
i2 2 i5
x–4 x–2 x–y 2 2
  0 2  2 
2 2 2
 3x – y = 6 ...(1) 4 2 2
At the junction 2 applying KCL, 4V 2V 2V
i4 + i5 + i6 = 0
y–x y–2 y O O O O
  0
2 2 2
 3y – x = 2 ...(2)
Solving (1) & (2)
9x – 3y = 18  3y – x = 2
 8x = 20
5 3
x= , y=
2 2
Just put the values of x & y & then the evaluate the current in every branch

394,50 - Rajeev Gandhi Nagar Kota, Ph. No. : 93141-87482, 0744-2209671


IVRS No : 0744-2439051, 52, 53, www. motioniitjee.com , info@motioniitjee.com

Downloaded from www.iitjeephysics4u.com


Page # 16 CURRENT ELECTRICITY

(b) The Kirchhoff's Voltage Law


The Kirchhoff's voltage law (KVL) states that the algebraic sum of the potential difference around any
closed loop of an electric circuit is zero. The KVL is a statement of conservation of energy. The KVL
reflects that electric force is conservative, the work done by a conservative force on a charge taken
around a closed path is zero.
We can move clockwise or anticlockwise, it will make no difference because the overall sum of the
potential difference is zero.
We can start from any point on the loop, we just have to finish at the same point.
An ideal battery is modelled by an independent voltage source of emf E and an internal resistance r as
shown in figure A real battery always absorbs power when there is a current through it, thereby
offering resistance to flow of current.
b c
Applying KVL around the single loop in anticlockwise
direction, starting from point A, we have

+ IR + Ir – E = 0 r
R
In the In the opposite From positive Real
opposite direction to to negative Battery
direction current terminal E
to current
a d
E
Hence, I
Rr

Ex.15 Find current in the circuit

1
20 V 15 V
4
2
10 V

3 30 V 20 V – i
D +
C
Sol.  all the elements are connected in series + 1 15 V
 current in all of them will be same 4i 4  –
– 2 2i
let current = i 10 V +
Applying kirchhoff voltage law in ABCDA loop 3 i
A + – B
10 + 4i – 20 + i + 15 + 2i – 30 + 3i = 0 i
3i 30 V
10 i = 25  i = 2.5 A

Ex.16 Find the current in each wire applying only kirchhoff voltage law
10 50V
F E D

2 2
1

30V
A B C

394,50 - Rajeev Gandhi Nagar Kota, Ph. No. : 93141-87482, 0744-2209671


IVRS No : 0744-2439051, 52, 53, www. motioniitjee.com , info@motioniitjee.com
Downloaded from www.iitjeephysics4u.com
CURRENT ELECTRICITY Page # 17

Sol. Applying kirchhoff voltage law in loop ABEFA


i1 + 30 + 2 (i1 + i2) – 10 = 0
3i1 + 2i2 + 20 = 0 ...(i)
Applying kirchhoff voltage law in BCDEB
+ 30 + 2(i1 + i2) + 50 + 2i2 = 0
4i2 + 2i1 + 80 = 0
2i2 + i1 + 40 = 0 ...(ii)
Solving (i) and (ii) 10V 50V
F E D
3[–40 –2i2] + 2i2 + 20 = 0 i1 i2 i2
i1
–120 – 4i2 + 20 = 0 +
– 2(i1+i2) 2  –
i2 = – 25 A 1 i1 – 2i 2 2
and i1 = 10 A + 30V +
i1 i1+i2 i2
 i1 + i2 = – 15 A
current in wire AF = 10 A from A to E A B i2 C
current in wire EB = 15 A from B to E
current in wire DE = 25 A from D to C

7. COMBINATION OF RESISTANCE
A number of resistance can be connected in a circuit and any complicated combination can be, in
general, reduced essentially to two different types, namely series and parallel combinations.
(a) Resistance in Series

R1 R2 R3 R
V1 V2 V3

V V

(i) In this combination the resistance are joined end to end. The second end of each resistance is
joined to first end of the next resistance and so on. A cell is connected between the first end of
first resistance and second end of last resistance. Figure shows three resistances R1, R2 and R3
connected in this way. Let V1, V2 and V3 are the potential differences across these resistances.
(ii) In this combination current flowing through each resistance will be same and will be equal to
current supplied by the battery.
(iii) As resistances are different and current flowing through them is same, hence potential differ-
ences across them will be different. Applied potential difference will be distributed among three
resistances directly in their ratio.
As i is constant, hence V  R
i.e., V1 = iR1, V2 = iR2, v3 = iR3
(iv) If the potential difference between the points A and D is V, then
V = V1 + V2 + V3 = i (R1 + R2 + R3)
(v) If the combination of resistances between two points is replaced by a single resistance R such
that there is no change in the current of the circuit in the potential difference between those
two points, then the single resistance R will be equivlaent to combination and V = i R i.e.,
iR = i (R1 + R2 + R3) or R = R1 + R2 + R3

394,50 - Rajeev Gandhi Nagar Kota, Ph. No. : 93141-87482, 0744-2209671


IVRS No : 0744-2439051, 52, 53, www. motioniitjee.com , info@motioniitjee.com

Downloaded from www.iitjeephysics4u.com


Page # 18 CURRENT ELECTRICITY

(v) Thus in series combination of resistances, important conclusion are


(a) Equivalent Resistance > highest individual resistance
(b) Current supplied by source = Current in each resistance
V V1 V2 V3
or R  R  R = R = R = R
1 2 3 1 2 3

(c) The total potential difference V between points A and B is shared among the three resistances
directly in their ratio
V1 : V2 : V3 = R 1 : R 2 : R 3

(b) Resistance in Parallel


i1 R1
i A i2 R2 B
i R
i3 R3

V
(i) When two or more resistance are combined in such a way that their first ends are connected to
one terminal of the battery while other ends are connected to other terminal, then they are
said to be connected in parallel. Figure shows three resistances R1, R2 and R3 joined in parallel
between two points A and B. Suppose the current flowing from the battery is i. This current
gets divided into three parts at the junction A. Let the currents in three resistance R1 , R2 and
R3, are i1, i2, i3 respectively.
(ii) Suppose potential difference between points A and B is V. Because each resistance is con-
nected between same two points A and B, hence potential difference across each resistance
will be same and will be equal to applied potential difference V.
(iii) Since potential difference across each resistance is same, hence current approaching the
junction A is divided among three resistances reciprocally in their ratio.
As V is constant, hence i  (1/R) i.e.,

V V V
i1  , i2  and i3 
R R2 R3
(iv) Because i the main current which is divided into three parts i1, i2 and i3 at the junction A.

 1 1 1 
hence, i  i1  i 2  i3  V  R  R  R 
 1 2 3

V
If the equivalent resistance between the points A and B is R, then i =
R

V  1 1 1  1 1 1 1
Thus, R  V  R  R  R  or R  R  R  R
 1 2 3  1 2 3

(v) Thus in parallel combination of resistance important conclusion are :


(a) Equivalent resistance < lowest individual resistance
(b) Applied potential difference = Potential difference across each resistance.
or iR = i1R1 = i2R2 = i3R3
(c) Current approaching the junction A = Current leaving the junction B and current is shared
among the three resistances in the inverse ratio of resistances

1 1 1
i1 : i2 : i3 = R : R : R
1 2 3

394,50 - Rajeev Gandhi Nagar Kota, Ph. No. : 93141-87482, 0744-2209671


IVRS No : 0744-2439051, 52, 53, www. motioniitjee.com , info@motioniitjee.com
Downloaded from www.iitjeephysics4u.com
CURRENT ELECTRICITY Page # 19

(i) If two or more resistance are joined in parallel then i1R = i1R2 = i3R3............
i.e., iR = constant i.e., a low resistance joined in parallel always draws a higher current.
(ii) When two resistance R1 and R2 are joined in parallel, then

i1R1 i12R12 i12R1t R2 H1 R 2


 1 or  1 or 2
 or H  R
i2R 2 i22R22 i2 R2t R1 2 1

i.e., heat produced will be maximum in the lowest resistance.

Ex.17 Find current which is passing through battery.


2

3

6

30V 1
Sol. Here potential difference across each resistor is not 30 V
 battery has internal resistance here the concept of combination of resistors is useful.
Req = 1 + 1 = 2

30
i  15 A
2
Ex.18 Find equivalent Resistance

A × × B
R R R

R R
Sol. A B
VA R VB VA VB

Here all the Resistance are connected between the terminals A and B. So, Modified circuit is

A R
R B
So Req=
3
R

Ex.19 Find the current in Resistance P if voltage supply between A and B is V volts
R

P R R
A B
R

3R
Sol. Req =
5

394,50 - Rajeev Gandhi Nagar Kota, Ph. No. : 93141-87482, 0744-2209671


IVRS No : 0744-2439051, 52, 53, www. motioniitjee.com , info@motioniitjee.com

Downloaded from www.iitjeephysics4u.com


Page # 20 CURRENT ELECTRICITY

R
A B
VA VB C
P
5V C
I= Modified circuit R R
3R A B
5V R
R 2V
Current in P  3R =
1.5R  R 3R

R R/2

P
A B
R

4 2

120V 6
Ex.20 Find the current in 2  resistance. 3 1

4 4

4
Sol. 2, 1 in series = 3
12A 12A
18
3, 6 in parallel =  2 2
9
2, 4 in series = 6
6, 3 is parallel = 2 4 12A
Req = 4 + 4 + 2 = 10 

120
i  12 A
10

8
So current in 2 Resistance = A
3
4 8
2 A A
12A 4A 3 3 3

3 6
8A

4

394,50 - Rajeev Gandhi Nagar Kota, Ph. No. : 93141-87482, 0744-2209671


IVRS No : 0744-2439051, 52, 53, www. motioniitjee.com , info@motioniitjee.com
Downloaded from www.iitjeephysics4u.com
CURRENT ELECTRICITY Page # 21

3
r r
1 6
r r r
r
A B
4 r
• SPECIAL PROBLEMS
r r r
2 7
r r
5

We wish to determine equivalent resistance between A and B. In figure shown points (1,2) (3, 4, 5) and (6,
7) are at same potential Equivalent circuit can be redrawn as in figure shown.
The equivalent resistance of this series combination is

r r r r 3r
R eq.     
2 4 4 2 2
r 1,2 r 3,4,5 r 6,7 r
r r
A B
r r r r

r r

In the figure shown, the resistances specified are in ohms. We wish to determine the equivalent resistance
between point A and D. Point B and C, E and F are the the same potential so the circuit can be redrawn
as in figure shown.
Thus the equivalent resistance is 1 .
A

2
D 2
1 1
1 1 1 2 1

E 1 F A 1 2 1
2 2 B,C E,F D
B C
1
In the network shown in figure shown all the resistances are equal, we wish to determine equivalent
resistance between A and E. Point B and D have same potential, similarly F and H have same potential.
The equivalent circuit is shown in figure shown. The equivalent resistance of network is 7R/2.
B

R R
R
F R
R R
R E G R R
A C
R R R R
R R E
A R R C R G R R
H B,D
R R F,H
R
R
D

394,50 - Rajeev Gandhi Nagar Kota, Ph. No. : 93141-87482, 0744-2209671


IVRS No : 0744-2439051, 52, 53, www. motioniitjee.com , info@motioniitjee.com

Downloaded from www.iitjeephysics4u.com


Page # 22 CURRENT ELECTRICITY

Ex.21 In the circuit shown in figure. (a) find the current flowing through the 100  resistor connecting
points U and S.
Sol. Figure (b) shows simplified circuit. The battery is directly attached to resistor 90hence current in it is
2 A, see figure (c), The total resistance of second branch is also 90 , hence current divides equally.
Now current through 45  resistor is 2 A and it is a combination of two equal 90  resistors. Once
again current divides equally. 90  resistor is a series combination of 40  and 50 , hence current
through them is equal, i.e.,

1 A. As 50  resistor is a parallel combination of two equal 100  resistors, they must have the same
current i.e., 0.5 A
8. WHEATSTONE'S BRIDGE B
i1 – ig
P ig Q
i1
A C
G
i2
K2 i2+ig
R S

i1+i2 D
i1+i2

+ –
K1
E

(i) Wheatstone designed a network of four resistances with the help of which the resistance of a given
conductor can be measured. Such a network of resistances is known as Wheastone's bridge.
(ii) In this bridge, four resistance P, Q, R and S are so connected so as to form a quadrilateral ABCD. A
sensitive galvanometer and key K2 are connected between diagonally opposite corners B and D, and a
cell and key K1 are connected between two other corners A and C (figure shown)
(iii) When key K1 is pressed, a current i flows from the cell. On reaching the junction A, the current i gets
divided into two parts i1 and i2. Current i1 flows in the arm AB while i2 in arm AD. Current i1, on reaching
the junction B gets further divided into two parts (i1 – ig) and ig, along branches BC and BD respectively.
At junction D, currents i2 and ig are added to give a current (i2 + ig), along branch DC. (i2 – ig) and (i2
+ ig) add up at junction C to give a current (i1 + i2) or i along branch CE. In this way, currents are
distributed in the different branches of bridge. In this position, we get a deflection in the galvanometer.
(iv) Now the resistance P,Q,R and S are so adjusted that on pressing the key K2, deflection in the galvanom-
eter becomes zero or current ig in the branch BD becomes zero. In this situation, the bridge is said to
be balanced.

394,50 - Rajeev Gandhi Nagar Kota, Ph. No. : 93141-87482, 0744-2209671


IVRS No : 0744-2439051, 52, 53, www. motioniitjee.com , info@motioniitjee.com
Downloaded from www.iitjeephysics4u.com
CURRENT ELECTRICITY Page # 23

(v) I n this balanced position of bridge, same current i1 flows in arms AB and BC and similarly same current i2
in arms AD and DC. In other words, resistances P and Q and similarly R and S, will now be joined in
series.
(vi) Condition of balance : Applying Kirchhoff's 2nd law to mesh ABDA, i1P + igG – i2R = 0...(1)
Similarly, for the closed mesh BCDB, we get, (i1 – ig) Q – (i2 + ig)S – igG = 0 ...(2) When
bridge is balanced, ig = 0. Hence eq. (1) & (2) reduce to
i1P – i2R = 0 or i1P = i2R ....(3)
i1Q – i2S = 0 or i1Q = i2S ...(4)

P R
Dividing (3) by (4), we have,  ....(5)
Q S
This is called as condition of balanced for Wheatstone's Bridge.
(vii) It is clear from above equation that if ratio of the resistance P and Q, and the resistance R are known,
then unknwon resistance S can be determined. This is the reason that arms P and Q are called as ratio
arms, arm AD as known arm and arm CD as unknown arm.

(viii) When the bridge is balanced then on inter-changing the positions of the galvanometer and the cell there
is no effect on the balance of the bridge. Hence the arms BD and AC are called as conjugate arms of
the bridge.
(ix) The sensitivity of the bridge depends upon the value of the resistances. The sensitivity of bridge is
maximum when all the four resistances are of the same order.
Ex.22 Find equivalent resistance of the circuit between the terminals A and B.

10 6

A 20 B

5 3

Sol. Since the given circuit is wheat stone bridge and it is in balance condition.
 10 × 3 =30 = 6× 5
hence this is equivalent to

16  8 16
R eq   
16  8 3
10 6

A B

5 3

Ex.23 Find the equivalent resistance between A and B

R R
A B
R

394,50 - Rajeev Gandhi Nagar Kota, Ph. No. : 93141-87482, 0744-2209671


IVRS No : 0744-2439051, 52, 53, www. motioniitjee.com , info@motioniitjee.com

Downloaded from www.iitjeephysics4u.com


Page # 24 CURRENT ELECTRICITY

Sol. This arrangement can be modified as shown in figure since it is balanced wheat stone bridge.

2R  2R
R eq  R
2R  2R

C
R R

A R B

R R
D

8.1 Unbalanced Wheatstone Bridge

10 B(x) 5
A C
i1 i5 i3
i i
v 2 0
i2 i4
D F
5 E(y) 10
Ex.24

v 0
v

Find equivalent resistance ?


Sol. Let potential at point B is x and E is Y

v
R eq 
i
Applying KCl at point B

x–v x–y x–0


  0
10 2 5
8x – 5y = v ...(1)
Applying KCL at point E

y–v y–x y–0


  0
5 2 10
 8y – 5x = 2v ...(2)

6v 7v
solving x & y x , y
13 13
current from branches BC & EF adds up to give total current (i) flowing in the circuit.

x–0 y–0 19v


i = i3 + i4 =  =
5 10 130

V 130
i  Req. =
 R eq 19

394,50 - Rajeev Gandhi Nagar Kota, Ph. No. : 93141-87482, 0744-2209671


IVRS No : 0744-2439051, 52, 53, www. motioniitjee.com , info@motioniitjee.com
Downloaded from www.iitjeephysics4u.com
CURRENT ELECTRICITY Page # 25

Ladder Problem :

R R R R R
A 

R R R R R

B 

Find the effective resistance between A & B ?


Sol. Let the effective resistance between A & B be RE since the network is infinite long, removal of one cell
from the chain will not change the network. The effective resistance between points C & D would also
be RE.
The equivalent network will be as shown below
R C
A

R RE

B D
The original infinite chain is equivalent to R in series with R & RE in parallel.

RRE
RE  R 
R  RE
RER + RE2 = R2 + 2RRE  RE2 – RRE – R2 = 0

R(1  5 )
RE 
2

R kR k2R k3R
A

R kR k2R k3R
Ex.25

B
Find the equivalent resistance between A & B ? R C
Sol. As moving rom one section to next one, resistance is increasing A

by k times. Since the network is infinitely long, removal of one R kRE


section from the chain will bring a little change in the network.
The effective resistance between points C & D would be kRE (where B
RE is the effective resistance) D
 Effective R between A & B.

R(kR E )
RE  R 
kR E  R
On solving we get

2kR – R  (R – 2kR)2  4kR 2


RE 
2k
394,50 - Rajeev Gandhi Nagar Kota, Ph. No. : 93141-87482, 0744-2209671
IVRS No : 0744-2439051, 52, 53, www. motioniitjee.com , info@motioniitjee.com

Downloaded from www.iitjeephysics4u.com


Page # 26 CURRENT ELECTRICITY

15. Symmetrical Circuits :


Some circuits can be modified to have simpler solution by using symmetry if they are solved by traditional
method of KVL and KCL then it would take much time.
Ex.26 Find the equivalent Resistance between A and B
C
R R
R
R R
A B

R
R R
D
Sol. I Method : MIRROR SYMMETRY
The branches AC and AD are symmetrical
 current through them will be same.
The circuit is also similar from left side and right side like mirror images with a mirror placed alone CD
therefore current distribution while entering through B and an exiting from A will be same. Using all
these facts the currents are as shown in the figure. It is clear that current in resistor between C and
E is 0 and also in ED is 0. It's equivalent is shown in figure (b)
C
R R
R R R
i i1 i–2i1 i1
A B R R
i–2i1 E i1 i
A B
i1 R R i R
R R
D

(fig.a) (fig.b)

2R
R eq 
3
II Method : FOLDING SYMMETRY
 The potential difference in R between (B, C) and between (B, D) is same VC = VD
Hence the point C and D are same hence circuit can be simplified as
R E R
A B
R R/2
R R

R R
C,D
This called folding.
Now , it is Balanced Wheatstone bridge
R E R
A B
R/2 R/2
R/2

(C,D)

2R  R 2R
R eq  
2R  R 3

394,50 - Rajeev Gandhi Nagar Kota, Ph. No. : 93141-87482, 0744-2209671


IVRS No : 0744-2439051, 52, 53, www. motioniitjee.com , info@motioniitjee.com
Downloaded from www.iitjeephysics4u.com
CURRENT ELECTRICITY Page # 27

In II Method it is not necessary to know the currents in CA and DA

Ex.27 Find the equivalent Resistance between A and B


C
R 2R
R
R 2R
A B
E
R
R 2R

D
Sol. In this case the circuit has symmetry in the two branches AC and AD at the input
 current in them are same but from input and from exit the circuit is not similar
( on left R and on right 2R) C
 on both sides the distribution of current will not be similar.
R 2R
Here Vc = Vd
R
hence C and D are same point x y
R 2R
So, the circuit can be simplified as A B
i i–2x E
y
Now it is balanced wheat stone bridge. R R 2R
x
R E 2R
A B D
R/2
R/2 R

C,D

3R 9
3R  R
R eq  2 2 R
3R = 9
3R 
2 2
Ex.28 Find the equivalent Resistance between A and B
R
R R R
R
R R
A B
R R
R R
R

A R B
R R R
R
x R R x
A x i–2x B
i R i–2x x
Sol.
R R R
C R D

Here VA = VC and VB = VD

394,50 - Rajeev Gandhi Nagar Kota, Ph. No. : 93141-87482, 0744-2209671


IVRS No : 0744-2439051, 52, 53, www. motioniitjee.com , info@motioniitjee.com

Downloaded from www.iitjeephysics4u.com


Page # 28 CURRENT ELECTRICITY

Here the circuit can be simplified as

y R R y
A B
i i–y i–y
R/2
R/2 R/2 R/2
R/2

R R
4R A B
2R 
R eq 3 4R R/2 R/2
10R = Ans. R/2 R/2
5
3 R/2
2R 2R
A B A B

R/2 R/2
4R/3
R/3
5 6
Ex.29 Twelve equal resistors each R  are connected to form the
edges of a cube. Find the equivalent resistances of
8 7
the network.
(a)When current enters at 1 & leaves at 6 (body diagonal)
Sol. Here 2, 4, 8 are equipotential points (if we move from 1 4 3
 2, 4, 8 it comes along the edge & 6  2, 4, 8 it comes
along face diagonal). Similarly 3, 5, 7 are equipotential 2
points. 1

1 6
2,4,8 3,5,7

R/3 R/6 R/3.

5R
R eq 
6
(b) When current enters at 1 and leaves at 2
Sol. Here 3, 7 are equipotential surface (if we move from 1  3, 7 we have along face and 2,  3, 7 we
move along edge) similarly 4, 8 are equipotential surface.
1 2

3, 7 4,8

5 6

394,50 - Rajeev Gandhi Nagar Kota, Ph. No. : 93141-87482, 0744-2209671


IVRS No : 0744-2439051, 52, 53, www. motioniitjee.com , info@motioniitjee.com
Downloaded from www.iitjeephysics4u.com
CURRENT ELECTRICITY Page # 29

7R
R eq 
12
(c) When current enters at 1 and leaves at 3
Sol. If we cut the cube along the plane passing through 2, 4, 5, 7 then by mirror symmetry, the final
configuration will be

5 6 5 6

8 7 8 7

3R
R eq 
4 3 4 3 4

2 2
1 1

9. COMBINATIONS OF CELLS
A cell is used to maintain current in an electric circuit. We cannot obtain a strong current from a single
cell. Hence need arises to combine two or more cells to obtain a strong current. Cells can be combined
in three possible ways :

(A) In series, (B) In parallel, and (C) In mixed grouping.


(A) Cells in Series

1 2 n nE
r r r nr
......
E E E I
I

R
R
In this combination, cells are so connected that –ve terminal of each cell is connected with the +ve
terminal of next and so on. Suppose n cells are connected in this way. Let e.m.f and internal resis-
tance of each cell are E and r respectively.
Net e.m.f of the cells = nE. Total internal resistance = nr. Hence total resistance of the circuit = nr +
R.

net e.m.f nE
If total current in the circuit is I, then I   ...(1)
Total Resistance nr  R

Case (i) : If nr < < R, then I  n E / R i.e., if total internal resistance of the cells is far less than external
resistance, then current obtained from the cells is approximately equal to n times the current ob-
tained from a single cell. Hence cells, whose total internal resistance is less than external resistance,
just be joined in series to obtain strong current.

nE E
Case (ii) : If nr >> R, then I   i.e., if total internal resistance of the cells is much greater than the
nr r
external resistance, then current obtained from the combination of n cells is nearly the same as
obtained from a single cell. Hence there is no use of joining such cells in series.

394,50 - Rajeev Gandhi Nagar Kota, Ph. No. : 93141-87482, 0744-2209671


IVRS No : 0744-2439051, 52, 53, www. motioniitjee.com , info@motioniitjee.com

Downloaded from www.iitjeephysics4u.com


Page # 30 CURRENT ELECTRICITY

(B) Cells in Parallel

1 E r

E E r/n
2 r

......
I
n E r
R
R

(I) When E.M.F's and internal resistance of all the cells are equal : In this combination, positive termi-
nals of all the cells are connected at one point and negative terminals at other point. Figure shown
such cells connected in parallel across some external resistance R. Let e.m.f and internal resistance of
each cell are E and r respectively.
Because all the cells are connected in parallel between two points, hence e.m.f of battery = E.
Total internal resistance of the combination of n cells = r/n
Because external resistance R is connected in series with internal resistance, hence total resistance of
the circuit = (r/n) + R
If current in external resistance is I, then

net E.M.F E nE
I = 
Total resistance (r / n)  R r  nR

nE E
Case (I) : If r << R, the I   i.e., if internal resistance of the cells is much less than external resis-
nR R
tance, then total current obtained from combination is nearly equal to current given by one cells
only. Hence there is no use of joining cells of low internal resistance in parallel.

nE
Case (II) : If r >> R, then I  i.e., if the internal resistance of the cells is much higher than the external
r
resistance, then total current is nearly equal to n times the current given by one cell. Hence cells of
high internal resistance must be joined in parallel to get a strong current.

(II) When emf's and internal resistance of all the cells connected in parallel are different.
In this case, total current in external resistance is obtained with the help of Kirchhoff's laws. Figure
shows three cells of e.m.f E1, E2 and E3 and internal resistances r1, r2 and r3 connected in parallel across
some external resistance R. Suppose currents given by three cells are i1, i2 and i3. Hence according to
Kirchhoff's first law, total current I in external resistance R, is given by
I = i1 + i2 + i3 ...(1) E1 r1
i1
Applying Kirchhoff's 2nd law to closed mesh ABEF we get F E
 (E1 – IR )  i2 E2 r2
IR + i1r1 = E1 or i1 =  r1
 ...(2) G D
 
Similarly, for closed meshes ABDG and ABCH, we get i3 E3 r3
H C
E 2 – IR
i2  ....(3) I
r2 R
A B
E – IR
and i3  2 ....(4)
r3

394,50 - Rajeev Gandhi Nagar Kota, Ph. No. : 93141-87482, 0744-2209671


IVRS No : 0744-2439051, 52, 53, www. motioniitjee.com , info@motioniitjee.com
Downloaded from www.iitjeephysics4u.com
CURRENT ELECTRICITY Page # 31

Substituting eq. (2), (3) and (4) in eq. (1), we have

E1 – IR E 2 – IR E 3 – IR E1 E 2 E 3 1 1 1
I      – IR   
r1 r2 r3 r1 r2 r3  r1 r2 r3 

  1 1 1  E E E (E1 / r1 )  (E 2 / r2 )  (E 2 / r3 )
or I 1  R     1  2  3 or I 
 r r
 1 2 r3 
 r1 r2 r3 1  R(1/ r1  1/ r2  1/ r3 )

Ei Ei
ri i
r i 1
If n cells are joined in parallel, then I  and Eeq.  , req. 
1 1 1
1 R r r
i i
r i i

(C) Cells in Mixed Grouping

(1) (1) (2) (n)


.......

(2)
.......
.....

.....
.......
(m)

In this combination, a certain number of cells are joined in series in various rows, and all such rows are
then connected in parallel with each other.
Suppose n cells, each of e.m.f E and internal resistance r, are connected in series in every row and m
such rows are connected in parallel across some external resistance R, as shown in figure.
Total number of cells in the combination = mn. As e.m.f. of each row = nE and all the rows are connected
in parallel, hence net e.m.f of battery = nE.
Internal resistance of each row = nr. As m such rows are connected in parallel, hence total internal
 nr 
resistance of battery =  
m

 nr  
Hence total resistance of the circuit =  m   R
  
If the current in external resistance is I, then

net e.m.f nE mnE


I  
Total resistance (nr / m)  R nr  mR

mnE
 2
 nr – mr   2 nmrR
It is clear from above equation that I will be maximum when
[(nr – mR)2 + 2  nmrR] is minimum.
This will be possible when the quantity [nr – mR]2 is minimum. Because this quantity is in square, it can
not be negative, hence its minimum value will be equal to zero, i.e.,

nr
mR = nr or R=
m
394,50 - Rajeev Gandhi Nagar Kota, Ph. No. : 93141-87482, 0744-2209671
IVRS No : 0744-2439051, 52, 53, www. motioniitjee.com , info@motioniitjee.com

Downloaded from www.iitjeephysics4u.com


Page # 32 CURRENT ELECTRICITY

i.e., In mixed grouping of cells, current in external resistance will be maximum when total inter-
nal resistance of battery is equal to external resistance.
Because power consumed in the external resistance or load = I2R, hence when current in load is
maximum, consumed power in it is also maximum, Hence consumed power in the load will
nr
also be maximum when R = .
m

mnE mnE nE mE
Imax  or  or
2mR 2nr 2R 2r

Ex.30 Find the current in the loop.


1 40v, 2 

4 2

15v, 1 10v
1
3 10 35 v, 5 
20v,1
Sol. The given circuit can be simplified as

35 35
i 
10  5 15

7 7
= A  I A
3 3

Ex.31 Find the emf and internal resistance of a single battery which is equivalent to a combination of
three batteries as shown in figure.

10V 2
6V 1

4V 2

10V 2
6V 1
B
Sol.
A
C 4V 2

Battery (B) and (C) are in parallel combination with opposite polarity. So, their equivalent

10 –4

BC  2 2  5 – 2  3V
1 1 1

2 2
rBC = 1 
6V 1 3V 1
Now, ABC  6 – 3  3 V
rABC = 2 Ans.

394,50 - Rajeev Gandhi Nagar Kota, Ph. No. : 93141-87482, 0744-2209671


IVRS No : 0744-2439051, 52, 53, www. motioniitjee.com , info@motioniitjee.com
Downloaded from www.iitjeephysics4u.com
CURRENT ELECTRICITY Page # 33

10. ELECTRICAL POWER


The energy liberated per second in a device is called its power, the electrical power P delivered by an
electrical device is given by
dq
P= V = VI
dt

Power consumed by a resistor.


V2
P = VI = I2R = watt
R
The power P is in watts when I is in amperes, R is in ohms and V is in volts.

The practical unit of power is 1 kW = 1000 W.


V2
The formula for power P = I2R = VI = is true only when all the electrical power is dissipated as heat
R
and not converted into mechanical work, etc. simultaneously.

If the current enters the higher potential point of the device then electric power is consumed by it (i.e.

acts as load). If the current enters the lower potential point then the device supplies power (i.e. acts

as source.)

(A). JOULE'S LAW OF ELECTRICAL HEATING

When an electric current flows through a conductor electrical energy is used in overcoming the

resistance of the wire. If the potential difference across a conductor of resistance R is V volt and if a
current of I ampere flows the energy expanded in time t seconds is given by

W = VIt joule
V2
= I2Rt joule = t
R
The electrical energy so expanded is converted into heat energy and this conversion is called the

heating effect of electric current.

The heat generated in joules when a current of I amperes flows through a resistance of R ohm for t

seconds is given by
I2Rt
H = I2Rt joule = cal.
4.2
This relation is known as Joule's law of electrical heating.

Ex.32 If bulb rating is 100 watt and 220 V then determine

(a) Resistance of filament

(b) Current through filament

(c) If bulb operate at 110 volt power supply then find power consume by bulb.

Sol. Bulb rating in 100 W and 220 V bulb means when 220 V potential difference is applied between the two

ends then the power consume is 100 W

Here V = 220

394,50 - Rajeev Gandhi Nagar Kota, Ph. No. : 93141-87482, 0744-2209671


IVRS No : 0744-2439051, 52, 53, www. motioniitjee.com , info@motioniitjee.com

Downloaded from www.iitjeephysics4u.com


Page # 34 CURRENT ELECTRICITY

P = 100
V2
 100
R
So R = 484 

Since Resistance depends only on material hence it is constant for bulb


V 220 5
I   Amp.
R 22  22 11
power consumed at 110 V
110  110
 power consumed =  25 W
484

Ex.33 In the following figure, grade the bulb in order of their brightness :

50W, 220V
100W, 220V
20W, 220V

B1 B2 B3

220V

2
Vrated
Sol. Prated 
R

2
Vrated
R
Prated

 R 3  R 2  R1
Power = i2R
As current passing through every bulb is same
 Brightness order is B3 > B2 > B1

B1 B2

B3

Ex.34

V
The above configuration shows three identical bulbs, Grade them in order of their brightness.

394,50 - Rajeev Gandhi Nagar Kota, Ph. No. : 93141-87482, 0744-2209671


IVRS No : 0744-2439051, 52, 53, www. motioniitjee.com , info@motioniitjee.com
Downloaded from www.iitjeephysics4u.com
CURRENT ELECTRICITY Page # 35

Sol. B1 & B2 withdraw less current as compared to B3 because in series they give 2R resistance where as R
is the resistance dut to B3.
Power = i2 R
 Brightness order : B3 > B2 = B1.

B6

B5 B4 B3

B1 B2
Ex.35

V
Grade the bulbs in order of their brightness (All bubls are identical)
B6
1
Sol. As i
R

3 1 B5 B3
i1 : i 2  : =6:5 i4 B4
5R 2R

6i 5i i1 i3
 i1  , i2  B1 B2
11 11
1 1 i2
i3 : i 4  : i
2R R = 1 : 2
As i3 + i4 = i1
V
2i 4i
 i3 ; i4 
11 11
power = i2R
 Order of Brightness : B5 > B1 = B2 > B6 > B4 = B3

(B) MAXIMUM POWER TRANSFER THEOREM


Let E be emf and r internal resistance of the battery. It is supplying current to an external resistance
R
E
current in circuit I =
(R  r )

The power absorbed by load resistor R is


2
 E 
P=IR =  2  R
R r 
For maximum power transfer we take the derivative
of P w.r.t R, set it equal to zero and solve the equation
for R.
dP
0
dR

394,50 - Rajeev Gandhi Nagar Kota, Ph. No. : 93141-87482, 0744-2209671


IVRS No : 0744-2439051, 52, 53, www. motioniitjee.com , info@motioniitjee.com

Downloaded from www.iitjeephysics4u.com


Page # 36 CURRENT ELECTRICITY

R
dP (R  r )2 – R[2(R  r )]
 E2 0
dR (R  r )2
Solving for R, we have
(R + r)2 – R (2) (R + r) = 0 E
(R + r) – 2R = 0 r battery
R=r
For a given real battery the load resistance maximizes the power if it is equal to the internal resistance
of the battery.
P
 E 
2 R
P(R)    R
Rr 
r R R

E
O r

The maximum power transfer theorem in general, holds for any real voltage source. The resitance R
may be a single resistor or R may be the equivalent resistance of a collection of resistors.

11. INTRUMENTS
(A) AMMETER
It is a device used to measure current and its always connected in series with the 'element' through
which current is to be meaured, e.g., in figure (A) ammeter A1 will measure the current (I1) through
resistance R1, A2 measures current (I2) through R2 and R3 while A, measures current I( I1 + I2).
Regarding an ammeter it is worth noting that :
(1) The reading of an ammeter is always lesser than actual current in the circuit, e.g., true current in
V
the resistance R in the circuit shown in figure (B) is I =
R
However, when an ammeter of resistance r is used to measure current as shown in figure (C), the
reading will be

V  V
I'   I  
(R  r )  R 

I
– A +
I V V
R1 I1
+A –
1 –
I I I' I' A r
+
I2

R2 I2 R3 R R
A2
+ –
(B) (C)
(A)

(2) Smaller the resistance of an ammeter more accurate will be its reading. An ammeter is said to be
ideal if its resistance (r) is zero. However, as practically r  0, ideal ammeter cannot be realised
in practice.

394,50 - Rajeev Gandhi Nagar Kota, Ph. No. : 93141-87482, 0744-2209671


IVRS No : 0744-2439051, 52, 53, www. motioniitjee.com , info@motioniitjee.com
Downloaded from www.iitjeephysics4u.com
CURRENT ELECTRICITY Page # 37

(3) To convert a galvanometer into an ammeter of a certain range say I, a small resistance S (called
shunt) is connected in parallel with the galvanometer so that the current passing through the
galvanometer of resistance G becomes equal to its full scale deflection value Ig. This is possible
only if

IgG = (I – Ig)S (i – iG ) S
Ig i ig
i.e., S = (I – I ) G G
g
external
resistance

Ammeter
Ex.36 What is the value of shunt which passes 10% of the main current through a galvanometer of
99 ohm ?
Rg
Sol. As in figure RgIg = (I – Ig)S I Ig
G
I  I 
 99   I – S
10  10 
(I–Ig) S
 S = 11 
For calculation it is simply a resistance
A

Resistance of ammeter
R G .S
RA 
RG  S
for S << RG  RA = S
Ex.37 Find the current in the circuit also determine percentage error in measuring in current through
an ammeter (a) and (b).

2 2

A 0.5 
10V 10V

(a) (b)
10
Sol. In A I   5A
2

10
In B I   4A
2 .5

i – i'
Percentage error is =  100 = 20% Ans.
i
Here we see that due to ammeter the current has reduced. A good ammeter has very low resistance as
compared with other resistors, so that due to its presence in the circuit the current is not affected.

394,50 - Rajeev Gandhi Nagar Kota, Ph. No. : 93141-87482, 0744-2209671


IVRS No : 0744-2439051, 52, 53, www. motioniitjee.com , info@motioniitjee.com

Downloaded from www.iitjeephysics4u.com


Page # 38 CURRENT ELECTRICITY

Ex.38 Find the reading of ammeter. Is this the current through 6  ?


3
A

6
18V 1

36
Sol. R eq   1  3
36
Current through battery

18
I  6A
3
So, current through ammeter
6
 6  4A
9
No, it is not the current through the 6 resistor.
Ideal ammeter is equivalent to zero resistance wire for calculation potential difference across
it is zero.
(B) VOLTMETER
It is a device used to measure potential difference and is

I
I
R1 R2 I' +
V v r
R R

V1 V2
+ – + – I

+ V –
(B)
(A)

always put in parallel with the 'circuit element' across which potential difference is to be measured e.g.,
in Figure (A) voltmeter V1 will measure potential difference across resistance R1, V2 across resistance
R2 and V across (R1 + R2) with V = V1 + V2
Regarding a voltmeter it is worth noting that :
(1)The reading of a voltmeter is always lesser than true value, e.g., if a current I is passing through a
resistance R [Fig. (B)], the true value V = IR. However, when a voltmeter having resistance r is
connected across R, the current through R will become
r V
I'  I and so V'  I' R 
(R  r ) [1  (R / r )]
and as voltmeter is connected across R its reading V' is lesser than V.
(2) Greater the resistance of voltmeter, more accurate will be its reading. A voltmeter is said to be
ideal if its resistance r is infinite, i.e., it draws no current from the circuit element for its operation.
Ideal voltmeter has been realised in practice in the form of potentiometer.
(3) To convert a galvanometer into a voltmeter of certain range say V, a high resistance R is connected
in series with the galvanometer so that current passing through the galvanometer of resistance G
becomes equal to its full scale deflection value Ig. This is possible only if

394,50 - Rajeev Gandhi Nagar Kota, Ph. No. : 93141-87482, 0744-2209671


IVRS No : 0744-2439051, 52, 53, www. motioniitjee.com , info@motioniitjee.com
Downloaded from www.iitjeephysics4u.com
CURRENT ELECTRICITY Page # 39

v
Ig

G
R

V
V = Ig (G + R) i.e., R –G
Ig

Ex.39 A voltmeter has a resistance of G ohm and range of V volt. Calculate the resistance to be used
in series with it to extend its range to nV volt.
V
Sol. Full scale current ig =
G
to change its range
V1 = (G +Rs) ig
V
 nV = (G + Rs)
G
Rs = G(n – 1) Ans.

Ex.40 Find potential difference across the resistance 300  in A and B.

200  200 

100 V 300  100 V 300  v 600 

(A) (B)

100
Sol. In (A) : Potential difference =  300 = 60 volt
200  300

100  300  600 


In (B) Potential difference =   50 volt
300  600  300  600 
200 
300  600
We see that by connected voltmeter the voltage which was to be measured has charged. Such voltmeters
are not good. If its resistance had been very large than 300  then it would have not affected the
voltage by much amount.
(C) METRE-BRIDGE
Metre-bridge is a sensitive device based on the principle of wheatstone-bridge, for the determination of
the resistance of a conductor (wire). Its sensitivity is much more than that of the post-office box.

394,50 - Rajeev Gandhi Nagar Kota, Ph. No. : 93141-87482, 0744-2209671


IVRS No : 0744-2439051, 52, 53, www. motioniitjee.com , info@motioniitjee.com

Downloaded from www.iitjeephysics4u.com


Page # 40 CURRENT ELECTRICITY

Metre-bridge is shown in figure AC is one metre long wire of manganin or constantan which is fixed along
a scale on a wooden base. The area of cross-section of the wire is same at all places. The ends A and
C of the wire are joined to two L-shaped copper strips carrying binding-screws as shown. In between
these strips, leaving a gap on either side, there is a third copper strip having three binding screws. The
middle screw D is connected to a sliding jockey B through a shunted - galvanometer G. The knob of the
jockey can be made to touch at any point on the wire.
To measure the unknown resistance, the connection as shown in figure are made.
A resistance R is taken out from the resistance box and the key K is closed. Now the jockey is slided
along the wire and a point is determined such that, on pressing the jockey on the wire at that point
there is no deflection in the galvanometer G. In this position the points B and D are at the same
potential. The point B is called 'null-point'. The lengths of both the parts AB and BC of the wire are read
on the scale. Suppose the resistance of the length AB of the wire is P and that of the length BC is Q.
Then, by the principle of Wheastone-bridge. We have,

P R

Q S
Let the length AB be l cm. Then the length BC will be = (100 – l) cm.

l
 resistance of AB, i.e. P = , and resistance of BC, Q =  (100 – l)/A
A
where  is the specific resistance of the material of the wire and 'A' is the area of cross-section of the
wire. Thus

P l
 ...(i)
Q (100 – l )

P
Substituting this value of in eq. (i), we get
Q

l R R(100 – l)
 or S
(100 – l ) S l
R is the resistance taken in the resistance box and l is the length measured. Hence, the value of
resistance S can be determined from the above formula.
A number of observations are taken for different resistances in the resistance box and for each
observation the value of S is calculated.
Finally, the experiment is repeated by interchanging the unknown resistances S and the resistance
box. The mean of the values of S is then obtained.
Ex.41 In a meter bridge experiment, the value of unknown resistance is 2. To get the balancing
point at 40 cm distance from the same end, then what will be the resistance in the resistance
box ?
Sol. Apply condition for balance wheat stone bridge,
P  P 100 – 40
  
Q 100 –  2 40
P=3 Ans.
(D) POTENTIOMETER
A potentiometer is used to compare e.m.fs. of two cells or to measure internal resistance of a cell.
Principle : The potentiometer is based upon the principle that when a constant current is passed
through a wire of uniform area of cross-section, the potential drop across any portion of the wire is
directly proportional to the length of that portion.

Construction : A potentiometer consists of a number of segments of wire of uniform area of cross


section stretched on a wooden board between two thick copper strips. Each segment of wire is 100 cm
long. The wire is usually of constantan or manganin. A metre rod is fixed parallel to its length. A battery

394,50 - Rajeev Gandhi Nagar Kota, Ph. No. : 93141-87482, 0744-2209671


IVRS No : 0744-2439051, 52, 53, www. motioniitjee.com , info@motioniitjee.com
Downloaded from www.iitjeephysics4u.com
CURRENT ELECTRICITY Page # 41

connected across the two end terminals sends current through the wire, which is kept constant by
using a rheostat.
Theory : Let V be potential difference across certain portion of wire, whose resistance is R. If I is the
current through the wire, then V = IR

l
We know that R =  ,
A
where l, A and  are length, area of cross-section and resistivity of the material of wire respectively.
l
 V  I
A
If a constant current is passed through the wire of uniform area of cross-section, then I and A are
constants. Since, for a given wire,  is also constant, we have
V = constant × l
or Vl
Hence, if a constant current flows through a wire of uniform area of cross-section, then potential drop
along the wire is directly proportional to the length of the wire.
Applications of a potentiometer. A potentiometer can be put to following uses :
1. To compare e.m.fs. of two cells : Two cells, whose e.m.fs. are E1 and E2, can be compared by making
use of the ciruit as shown in figure. The positive poles of both the cells are connected to the terminal
A of the potentiometer. The negative poles of the two cells are connected to terminals 1 and 2 of a
two way key. while its common terminal is connected to a jockey j through a galvanometer G. An
auxiliary or driver battery of e.m.f E, an ammeter A, rheostat Rh and a one way key K are connected
between the end terminals A and B of the potentiometer. Thus, the positive poles of the two cells as
well as the positive pole of auxiliary battery are connected at the common point A. It may be pointed
that the e.m.f of auxiliary battery should always be greater than the e.m.f of either of the two cells.
K
– + + E' –
A
Rh

B
200 300
200

100
A

E1 1
+ –
G
+ –
3
E2
2
To compare the e.m.fs of the two cells, a constant current is passed through the potentiometer wire
between points A and B. The current is kept constant by using the rheostat.
When the plug is put in the gap between the terminals 1 and 3 of the two way key, the cell of e.m.f. E1
will come in the ciruit. Suppose the balancing length (between points A and J) is l1. If x is the
resistance per unit length of the potentiometer wire and I, the constant current flowing through it,
then

394,50 - Rajeev Gandhi Nagar Kota, Ph. No. : 93141-87482, 0744-2209671


IVRS No : 0744-2439051, 52, 53, www. motioniitjee.com , info@motioniitjee.com

Downloaded from www.iitjeephysics4u.com


Page # 42 CURRENT ELECTRICITY

E1 = (xl1) I
When the key is put in the gap between the terminals 2 and 3 and removed from the gap between 1
and 3, the cell of e.m.f E2 wil be included in the circuit. Let the balancing length be l2 in this case.
Then,
E2 = (x l2) I
Dividing above equation
E1 l1

E 2 l2
Note : It may be pointed out that the e.m.f of auxiliary battery should always be greater than the
e.m.f. of the either of the two cells.
2. To measure internal resistance of a cell. The internal resistance of a cell may be found by using a
potentiometer by setting up the circuit as shown in figure.
A constant current I is maintained through the potentiometer wire with the help of the rheostat.
Plug in the key K2 is kept out and the jockey is moved over the potentiometer wire so as to balance the
e.m.f. E of the cell, whose internal resistance is to be found. Let l1 be the balancing length of the
potentiometer wire between point A and jockey J. If x is resistance per unit length of the wire, then
E = (x l1) I
Rh
– + + E' –
A K1

B
J

E
G R
+ –

S
K2
With the help of resistance box S, introduce resistance say S and then put the plug in key K2. Now find
the balance point for the terminal potential difference V between the two poles of the cell. If l2 is the
balancing length, then
V = (x l2) I
Dividing above equation , we have
E1 l1

E 2 l2
The internal resistance* of the cell is given by
E 
r   – 1 S
V 
Using above equation , we have

l  l1 – l2
r   1 – 1 S or r S
l
 2  l2

Knowing the values of l1, l2 and S, the internal resistance r of the cell can be found.
Note : Apart from uses, a potentiometer can be used to compare unknown resistances and to calibrate
a voltmeter or an ammeter.
We use potentiometer for two tasks :
(i) to find emf of a cell
(ii) to find internal resistance of a celll
We will first analyse the first task  to find emf of a cell through some examples

394,50 - Rajeev Gandhi Nagar Kota, Ph. No. : 93141-87482, 0744-2209671


IVRS No : 0744-2439051, 52, 53, www. motioniitjee.com , info@motioniitjee.com
Downloaded from www.iitjeephysics4u.com
CURRENT ELECTRICITY Page # 43

50V

Ex.42
10m
A
x
G
r
20V
Find the value of x if A is the null point ?

50
O
50V

Sol.
10m
50 O

30 G

20V
50
Potential gradient = = 5 V/m
10
 For 20V potential difference
20
  4m  x  10  4  6m
5
25 

200V

75 
10m
P
Ex.43 x
200V
25 
2 G 150 O
–50
120V
Find the value of x if P is a null point.
75 
Sol. Current in the main circuit 10m
P
200 – 25 i – 75 i = 0 150 O
x
i=2A
G
150 V
Potential gradient = 10 m  15 V / m
120V 30

394,50 - Rajeev Gandhi Nagar Kota, Ph. No. : 93141-87482, 0744-2209671


IVRS No : 0744-2439051, 52, 53, www. motioniitjee.com , info@motioniitjee.com

Downloaded from www.iitjeephysics4u.com


Page # 44 CURRENT ELECTRICITY

120 V / m
For 120 V   8m
15 V
 x = 10 – 8 = 2m E1
O E1
Now we will analyse the other task
to find internal resistance of the cell
using potentiometer.
The main key point is that first analyse the

main circuit then the auxillary circuit x P
O E1
(supplementary circuit)
G null deflection
E1 E
Potential gradient = r
 ER
Now for the auxillary circuit E Rr

E
i
Rr R
Er ER ER E1
E  ir  E     x
Rr Rr Rr 
Let we take some examples to understand the topic in better way.

90V 10

10cm 20
P

x
Ex.44
G
2

20V 2

Find x if P is a null point?


Sol. First analysing the main circuit, 90V 10
90 – 10i1 – 20 i1 = 0 0 60
i1 = 3 A. 90

60 V 10cm
Potential gradient = = 6V/m. 20
10m
0
Now analysing the auxillary circuit 60
G
20 – 2i2 – 2i2 = 0
i2 = 5 A. 20 2 10
0
10 5 20V 2
For 10 Ve  = m
6 3

5 25
 x = 10 – = m
3 3

394,50 - Rajeev Gandhi Nagar Kota, Ph. No. : 93141-87482, 0744-2209671


IVRS No : 0744-2439051, 52, 53, www. motioniitjee.com , info@motioniitjee.com
Downloaded from www.iitjeephysics4u.com
CURRENT ELECTRICITY Page # 45

100V

5

10m 20
0 P 80V
x
G
Ex.45 2V 2

8V 2

Find x if P is a null point ?


Sol. 100 – 5i1 – 20 i1 = 0 100V
i1 = 4 A 0 80
100 5
80
Potential gradient = = 8V/m
10 10m
8 – 2i2 – 2i2 – 2 = 0
0 P 80
3 G
i2 = A
2 20
2V 2 2
5v 5 0 5
for 5 volts  = m
8v / m 8 8
0
5 8V 2 5
 x= m
8

394,50 - Rajeev Gandhi Nagar Kota, Ph. No. : 93141-87482, 0744-2209671


IVRS No : 0744-2439051, 52, 53, www. motioniitjee.com , info@motioniitjee.com

Downloaded from www.iitjeephysics4u.com


Page # 46 CURRENT ELECTRICITY

Exercise - I (Objective Problems)


1. A wire of cross-section area A, length L1, resistiv- 6. Two wires each of radius of cross section r but of
ity 1 and temperature coefficient of resistivity 1 is different materials are connected together end to end
connected to a second wire of length L2, resistivity (in series). If the densities of charge carriers in the
2, temperature coefficient of resistivity 2 and the
two wires are in the ratio 1 : 4, the drift velocity of
same area A, so that wire carries same current. Total
electrons in the two wires will be in the ratio.
resistance R is independent of temperature for small
temperature change if (Thermal expansion effect is (A) 1 : 2 (B) 2 : 1
negligible) (C) 4 : 1 (D) 1 : 4
(A) 1 = –2 (B) 1L11 + 2L22 = 0 7. In a wire of cross-section radius r, free electrons
(C) L11 + L22 = 0 (D) None travel with drift velocity v when a current  flows
2. In order to increase the resistance of a given wire through the wire. What is the current in another wire
of uniform cross section to four times its value, a of half the radius and of the same material when the
fraction of its length is stretched uniformly till the full drift velocity is 2v ?
3 (A) 2I (B) I
length of the wire becomes times the original length
2 (C) I/2 (D) I/4
what is the value of this fraction ?
8. Read the following statements carefully :
1 1
(A) (B) Y : The resistivity of a semiconductor decreases with
4 8
increases of temperature.
1 1
(C) (D) Z : In a conducting solid, the rate of collision between
16 6
free electrons and ions increases with increase of tem-
3. A conductor with rectangular cross section has perature.
dimensions (a × 2a × 4a) as shown in figure. Resis-
Select the correct statement from the following
tance across AB is x, across CD is y and across EF is
(A) Y is true but Z is false
z. Then
(B) Y is false but Z is true
C F (C) Both Y and Z are true
(D) Y is true and Z is the correct reason for Y

2a 9. A piece of copper and another of germanium are


B
A 4a a cooled from room temperature to 80K. The resistance
of
(A) each of them increases
E D (B) each of them decreases
(A) x = y = z (B) x > y > z (C) copper increases and germanium decreases
(C) y > z > x (D) x > z > y (D) copper decreases and germanium increases
4. A brass disc and a carbon disc of same radius are 10. An insulating pipe of cross-section area ‘A’ contains
assembled alternatively to make a cylindrical conduc- an electrolyte which has two types of ions  their charges
tor. The resistance of the cylinder is independent of being –e and +2e. A potential difference applied be-
the temperature. The ratio of thickness of the brass tween the ends of the pipe result in the drifting of the
disc to that of the carbon disc is [ is temperature two types of ions, having drift speed = v (–ve ion) and
coefficient of resistance & Neglect linear expansion] v/4 (+ ve ion). Both ions have the same number per unit
 C C  C B volume = n. The current flowing through the pipe is
(A)   (B)   (A) nev A/2 (B) nev A/4
B B B C
(C) 5nev A/2 (D) 3nev A/2
 B C  B B 11. Current density in a cylindrical wire of radius R is
(C)   (D)  
C B C C
 x  R
5. A current of (2.5 ± 0.05) A flows through a wire J0   1 for 0  x 
 R 2
and develops a potential difference of (10 ± 0.1) volt. given as J   . The current
Resistance of the wire in ohm, is J x R
for  x  R
(A) 4 ± 0.12 (B) 4 ± 0.04  0 R 2
(C) 4 ± 0.08 (D) 4 ± 0.02
flowing in the wire is

394,50 - Rajeev Gandhi Nagar Kota, Ph. No. : 93141-87482, 0744-2209671


IVRS No : 0744-2439051, 52, 53, www. motioniitjee.com , info@motioniitjee.com
Downloaded from www.iitjeephysics4u.com
CURRENT ELECTRICITY Page # 47

7 1 17. A rectangular carbon block has dimensions 1.0


(A) J0 R2 (B) J0 R2
24 6 cm × 1.0 cm × 50 cm. Resistances are measured,
first across two square ends and then across two
7 5
(C) J0 R 2 (D) J0 R 2 rectangular ends, respectively. If resistivity of carbon
12 12 is 3.5 × 10–5 -m, then values of measured resistances
respectively are :
12. A current I flows through a uniform wire of diam-
eter d when the mean electron drift velocity is V. the
same current will flow through a wire of diameter d/2
made of the same material if the mean drift velocity
of the electron is
(A) v/4 (B) v/2 50 cm
(C) 2v (D) 4v 1cm

1cm
13. The current in a metallic conductor is plotted 35 2 5
(A)  10 , 7  10 
against voltage at two different temperatures T1 and 2
T2. Which is correct
5 15
T1 (B) 7  10  ,  10 2 
Current

2
T2 35
(C)  10  4 , 7  10 7 
2
15
Voltage (D) , 7  10 2 
2
(A) T1 > T2 (B) T1 < T2
(C) T1 = T2 (D) none
18. A storage battery is connected to a charger for
charging with a voltage of 12.5 Volts. The internal
14. A uniform copper wire carries a current i amperes
resistance of the storage battery is 1 . When the
and has p carriers per metre3. The length of the wire charging current is 0.5 A, the emf of the storage bat-
is  metres and its cross-section area is s metre2. If tery is :
the charge on a carrier is q coulombs, the drift veloc- (A) 13 Volts (B) 12.5 Volts
ity in ms–1 is given by (C) 12 Volts (D) 11.5 Volts
(A) i/  sq (B) i/psq
(C) psq/i (D) i/ps  q 19. The terminal voltage across a battery of emf E
can be
 (A) 0 (B) > E
15. In the presence of an applied electric field (E ) in (C) < E (D) all of above
a metallic conductor.

(A) The electrons move in the direction of E 20. In order to determine the e.m.f of a storage bat-
 tery it was connected in series with a standard cell in
(B) The electrons move in a direction opposite to E
(C) The electrons may move in any direction ran- a certain circuit and a current I1 was obtained. When
 the battery is connected to the same circuit opposite
domly, but slowly drift in the direction of E
to the standard cell a current I2 flow in the external
(D) The electrons move randomly but slowly drift in a
 circuit from the positive pole of the storage battery
direction opposite to E was obtained. What is the e.m.f 1 of the storage
battery? The e.m.f of the standard cell is 2.
16. A wire has a non-uniform cross-section as shown
in figure. A steady current flows through it. The drift I1  I2 I1  I2
(A)  1  I – I  2 (B)  1  I – I  2
speed of electrons at points P and q is vp and vQ. 1 2 2 1

I1 – I2 I2 – I1
(C)  1  I  I  2 (D)  1  I  I  2
1 2 1 2
P Q
21. One end of a Nichrome wire of length 2L and
(A) vp = vQ (B) vp < vQ cross-sectional area A is attatched to an end of an-
(C) vp > vQ (D) Data insufficient other Nichrome wire of length L and cross-sectional
area 2A. If the free end of the longer wire is at an
394,50 - Rajeev Gandhi Nagar Kota, Ph. No. : 93141-87482, 0744-2209671
IVRS No : 0744-2439051, 52, 53, www. motioniitjee.com , info@motioniitjee.com

Downloaded from www.iitjeephysics4u.com


Page # 48 CURRENT ELECTRICITY

electric potential of 8.0 volts, and the free end of the shown. What is the internal resistance of the cell ?
shorter wire is at an electric potential of 1.0 volt, the
potential at the junction of the two wires is equal to V(V)
C
(A) 2.4 V (B) 3.2 V
(C) 4.5 V (D) 5.6 V
22. In the figure shown, battery 1 has emf = 6V and V
internal resistance = 1. Battery 2 has emf = 2V and
internal resistance = 3. The wires have negligible
A
resistance. What is the potential difference across
the terminals of battery 2? I(A)
1
(A) x (B) y
1
(C) x/y (D) y/x
26. A cell of emf E has an internal resistance r & is
3 connected to rheostat. When resistance R of rheo-
stat is changed correct graph of potential difference
2 across it is
(A) 4V (B) 1.5V
(C) 5 V (D) 0.5V
V V
r (A) (B)
23.
 i R R
(a)
r V V
r

i  (No current) (C) (D)
(b) (b) R R
r 27. A circuit is comprised of eight identical batteries
 and a resistor R = 0.8. Each battery has an emf of
(b) 1.0 V and internal resistance of 0.2. The voltage
difference across any of the battery is
In which of the above cells, the potential difference
between the terminals of a cell exceeds its emf.
(A) a (B) b
(C) c (D) d

24. Under what condition current passing through the


resistance R can be increased by short circuiting the
battery of emf E2. The internal resistances of the two (A) 0.5 V (B) 1.0 V
batteries are r1 and r2 respectively. (C) 0 V (D) 2 V
E1 r1 E2 r2
28. In the circuit shown, what is the potential differ-
ence VPQ ?
Q
R
2V 4V
(A) E2r1 > E1(R + r2) (B) E1r2 > E2(R + r1)
1V
(C) E2r2 > E1(R + r2) (D) E1r1 > E2 (R + r1)
P
25. The diagram besides shows a circuit used in an (A) + 3V (B) + 2V
experiment to determine the emf and internal resis- (C) – 2V (D) none
tance of the cell C. A graph was plotted of the poten-
tial difference V between the terminals of the cell 29. In the circuit shown in figure reading of voltmeter
against the current I, which was varied by adjusting is V1 when only S1 is closed, reading of voltmeter is V2
the rheostat. The graph is shown on the right; x and when only S2 is closed. The reading of voltmeter is V3
y are the intercepts of the graph with the axes as when both S1 and S2 are closed then

394,50 - Rajeev Gandhi Nagar Kota, Ph. No. : 93141-87482, 0744-2209671


IVRS No : 0744-2439051, 52, 53, www. motioniitjee.com , info@motioniitjee.com
Downloaded from www.iitjeephysics4u.com
CURRENT ELECTRICITY Page # 49

3R
R a b
6R S1
3V,1 15V,2
S2
V

R
E
(A) V2 > V1 > V3 (B) V3 > V2 > V1 (A) 5 (B) 7
(C) V3 > V1 > V2 (D) V1 > V2 > V3 (C) 3 (D) 1

30. A 5 V battery with internal resistance 2  and a 34. The battery in the diagram is to be charged by
2V battery with internal resistance 1  are connected the generator G. The generator has a terminal volt-
to a 10  resistor as shown in the figure. age of 120 volts when the charging current is 10 am-
peres. The battery has an emf of 100 volts and an
P2
internal resistance of 1 ohm. In order to charge the
battery at 10 amperes charging current, the resis-
tance R should be set at
2V R
5V 10 
+
2 G
1

– +
100 V, 1
P1 (A) 0.1 (B) 0.5 
The current in the 10  resistor is : (C) 1.0  (D) 5.0 
(A) 0.03 A P1 to P2 (B) 0.03 A P2 to P1
(C) 0.27 A P1 to P2 (D) 0.27 A P2 to P1 35. ABCD is a square where each side is a uniform
wire of resistance 1. A point E lies on CD such that if
31. The Kirchhoff's first law (  i  0) and second law a uniform wire of resistance 1 is connected across
AE and constant potential difference is applied across
(  i R  0   E) , where the symbols have their usual A and C then B and E are equipotential.
meanings, are respectively based on
(A) conservation of charge, conservation of energy 1
A B
(B) conservation of charge, conservation of momen-
tum 1
(C) conservation of energy, conservation of charge 1 1
(D) conservation of momentum, conservation of charge
E
D C
32. In the network shown the potential difference 1
between A and B is (R = r1 = r2 = r3 = 1 , E1 = 3 V, E2 =
2 V, E3 = 1 V) CE CE
(A) 1 (B) 2
r1 E1 ED ED

E2 CE 1 CE
R r2 (C) ED  (D)  2
A 2 ED
B
36. In the given circuit the current flowing through
r3 E3 the resistance 20 ohms is 0.3 ampere while the
(A) 1 V (B) 2 V ammeter reads 0.8 ampere. What is the value of R1 ?
(C) 3 V (D) 4 V R1

20
33. Two batteries one of the emf 3V, internal resis- A
tance 1 ohm and the other of emf 15 V, internal resis-
15
tance 2 ohm are connected in series with a resis-
tance R as shown. If the potential difference between (A) 30 ohms (B) 40 ohms
a and b is zero the resistance of R in ohm is (C) 50 ohms (D) 60 ohms

394,50 - Rajeev Gandhi Nagar Kota, Ph. No. : 93141-87482, 0744-2209671


IVRS No : 0744-2439051, 52, 53, www. motioniitjee.com , info@motioniitjee.com

Downloaded from www.iitjeephysics4u.com


Page # 50 CURRENT ELECTRICITY

37. Find the current flowing through the resistance R1


39. The equivalent resistance between points A and
of the circuit shown in figure if the resistance are
B is :
equal to R1 = 10 , R2 =20 , and R3 = 30 , and the
potential of points 1, 2 and 3 are equal to 1  10 V, A
2 15
 2  6 V and 3  5 V . B
2
R2
8 10
1 R1 O

R2

20 10
3

(A) 0.1 A (B) 0.2 A 30 40


(C) 0.3 A (D) 0.4 A
65 45
(A)  (B) 
38. Two current elements P and Q have current volt- 2 2
age characteristics as shown below : 5 91
(C)  (D) 
2 2
P
40. The resistance of the series combination of two
1 resistances is S. Whey they are joined in a parallel,
the total resistance is P. If S = nP, then the minimum
10 P.D. (Volt) possible value of n is :
(A) 4 (B) 3
(C) 2 (D) 1
Q
41.Consider an infinte ladder network shown in figure.
1 A voltage V is applied between the points A and B.
This applied value of voltage is halved after each sec-
tion.
10 P.D. (Volt)
R1 R1 R1 R1 R1
Which of the graphs given below represents current A
voltage characteristics when P and Q are in series. R2 R2 R2 R2 R2
2 2
R
1 1 (A) R1/R2 = 1 (B) R1/R2 = 1/2
(A) (B) (C) R1/R2 = 2 (D) R1/R2 = 3
10 20 10 20
P.D.(Volt) P.D.(Volt)
42. The total current supplied to the circuit by the
battery is :
2 2

1 1
2
(C) (D) 6
10 20 10 20
P.D.(Volt) P.D.(Volt) 6V 3

1.5
2

1
(E)
(A) 1 A (B) 2 A (C) 4 A (D) 6 A
10 20
P.D.(Volt)

394,50 - Rajeev Gandhi Nagar Kota, Ph. No. : 93141-87482, 0744-2209671


IVRS No : 0744-2439051, 52, 53, www. motioniitjee.com , info@motioniitjee.com
Downloaded from www.iitjeephysics4u.com
CURRENT ELECTRICITY Page # 51

43. In the figure shown the current flowing through 2 be increased by 51 to secure balance. The unknown
R is : resistance in the fourth arm is

P Q
A R 2R R B

S 625

(A) from left to right (B) from right to left


(C) no current (D) None of these (A) 625  (B) 650 
(C) 676  (D) 600 
44. In the diagram resistance between any two junc-
tions is R. Equivalent resistance across terminals A 48. The current I drawn from the 5 volt source will be
and B is :
10 

5 10  20 

A B 10 
11R 18R I
(A) (B)
7 11
7R 11R + –
(C) (D)
11 18
5 volt
(A) 0.67 A (B) 0.17 A
45. In a balanced wheat stone bridge, current in the (C) 0.33 A (D) 0.5 A
galvanometer is zero. It remains zero when
(1) battery emf is increased 49.In a Wheat stone's bridge, three resistances P, Q
(2) all resistances are increased by 10 ohms and R are connected in the three arms and the fourth
(3) all resistances are made five times arm is foremd by two resistances S1 and S2 connected
(4) the battery and the galvanometer are interchanged in parallel. The condition for the bridge to be balanced
(A) only (1) is correct will be
(B) (1), (2) and (3) are correct
(C) (1), (3) and (4) are correct P R(S1  S 2 ) P R
(D) (1) and (3) are correct (A) Q  2S S (B) Q  S  S
1 2 1 2

46. A 3 volt battery with negligibel internal resistance P 2R P R ( S1  S 2 )


is connected in a circuit as shown in the figure. Cur- (C) Q  S  S (D) Q  S1S 2
1 2
rent i will be :
i 50.If the reading of ammeter A3 in figure is 0.75 A.
Neglecting the resistance of the ammeters, the read-
ing of ammeter A2 will be :
3 15
3V 3 A2

20
A1
3
(A) 1/3 A (B) 1 A
A3
(C) 1.5 A (D) 2 A
60
47. A Wheatstone’s bridge is balanced with a resis-
(A) 1.5 A (B) 3 A
tance of 625  in the third arm, where P, Q and S are
(C) 4.5 A (D) 6 A
in the 1st, 2nd and 4th arm respectively. If P and Q are
interchanged, the resistance in the third arm has to

394,50 - Rajeev Gandhi Nagar Kota, Ph. No. : 93141-87482, 0744-2209671


IVRS No : 0744-2439051, 52, 53, www. motioniitjee.com , info@motioniitjee.com

Downloaded from www.iitjeephysics4u.com


Page # 52 CURRENT ELECTRICITY

51.The resistance of all the wires between any two 57. A wire when connected to 220 V mains supply has
adjacent dots is R. Then equivalent resistance be- power dissipation P1. Now the wire is cut into two
tween A and B as shown in figure is : equal pieces which are connected in parallel to the
A same supply. Power dissipation in this case is P2. Then
P2 : P1 is :
(A) 1 (B) 4 (C) 2 (D) 3

B 58. Two bulbs rated (25 W – 220 V) and (100 W – 220


(A) 7/3 R (B) 7/6 R V) are connected in series to a 440 V line. Which one
(C) 14/8 R (D) None of these is likely to fuse ?
(A) 25 W bulb (B) 100 W bulb
52. In the box shown current i enters at H and leaves (C) both bulbs (D) none
at C.
i 2i i i i 59. Rate of dissipation of Joule’s heat in resistance
If i AB  , iDC  , iHA  , iGF  , iHE  , choose the
6 3 2 6 6 per unit volume is (symbols have usual meaning)
branch in which current is zero (A) E (B) J (C) JE (D) None
B C
60. The charge flowing through a resistance R varies
A
D with time as Q = 2t – 8t2. The total heat produced in
G 1
F the resistance is (for 0  t  )
H 8
i E R R
(A) joules (B) joules
(A) BG (B) FC (C) ED (D) none 6 3
R
53 A resistor of resistance R is connected to a cell of (C) joules (D) R joules
internal resistane 5 . The value of R is varied from 1 2
 to 5 . The power consumed by R :
61. If the length of the filament of a heater is re-
(A) increases continuously
duced by 10%, the power of the heater will
(B) decreases continuously
(A) increase by about 9%
(C) first decreases then increases
(B) increase by about 11%
(D) first increases then decreases.
(C) increase by about 19%
54. Power generated across a uniform wire connected (D) decrease by about 10%
across a supply is H. If the wire is cut into n equal
parts and all the parts are connected in parallel across 62. A heater A gives out 300 W of heat when con-
the same supply, the total power generated in the nected to a 200 V d.c. supply. A second heater B
wire is gives out 600 W when connected to a 200 v d.c.
H supply. If a series combination of the two heaters is
(A) 2 (B) n2H connected to a 200 V d.c. supply the heat output will
n
H be
(C) nH (D) (A) 100 W (B) 450 W
n
(C) 300 W (D) 200 W
55.A constant voltage is applied between the two
ends of a uniform metallic wire. Some heat is devel-
oped in it. The heat developed is doubled if 63. Two bulbs one of 200 volts, 60 watts & the other
(A) both the length and the radius of the wire are halved of 200 volts, 100 watts are connected in series to a
(B) both the length and the radius of the wire are doubled 200 volt supply. The power consumed will be
(C) the radius of the wire is doubled (A) 37.5 watt (B) 160 watt
(D) the length of the wire is doubled (C) 62.5 watt (D) 110 watt

56. When electric bulbs of same power, but different 64. Three 60 W light bulbs are mistakenly wired in
marked voltage are connected in series across the series and connected to a 120 V power supply. As-
power line, their brightness will be sume the light bulbs are rated for single connection to
(A) proportional to their marked voltage 120 V. With the mistaken connection, the power dissi-
(B) inversely proportional to their marked voltage pated by each bulb is
(C) proportional to the square of their marked voltage (A) 6.7 W (B) 13.3 W
(D) inversely proportional to the square of their marked (C) 20 W (D) 40 W
voltage
(E) the same for all of them

394,50 - Rajeev Gandhi Nagar Kota, Ph. No. : 93141-87482, 0744-2209671


IVRS No : 0744-2439051, 52, 53, www. motioniitjee.com , info@motioniitjee.com
Downloaded from www.iitjeephysics4u.com
CURRENT ELECTRICITY Page # 53

65. The ratio of powers dissipatted respectively in R P P


and 3R, as shown is R
(A) (B)

I I
3R P P

2R
(C) (D)
(A) 9 (B) 27/4
(C) 4/9 (D) 4/27 I I
66. If in the circuit, power dissipation is 150 W then R 70. If X, Y and Z in figure are identical lamps, which
is of the following changes to the brightnesses of the
R lamps occur when switch S is closed ?

z
x s
2

Y
(A) X stays the same, Y decreases
15V (B) X increases, Y decreases
(A) 2  (B) 6 
(C) X increases, Y stays the same
(C) 5  (D) 4 
(D) X decreases, Y increases
67.In the circuit shown, the resistances are given in
ohms and the battery is assumed ideal with emf equal 71. A battery consists of a variable number n of iden-
to 3.0 volts The resistor that dissipates the most tical cells having internal resistance connected in se-
power is ries. The terminals of the battery are short circuited
50 and the current I measured.
Which one of the graph below shows the relationship
R1 R3 R4 30 between I and n ?
60
3V R2 50
I/A

I/A
(A) (B)
(A) R1 (B) R2 n n
O O
(C) R3 (D) R4

68. What amount of heat will be generated in a coil of


I/A

I/A

resistance R due to a charge q passing through it if


the current in the coil decreases to zero uniformly (C) (D)
during a time interval t
n n
4 q2R q2R O O
(A) (B) ln
3 t 2t
2q2R ( 2 t )
(C) (D) ln 2
qR
I/A

3 t
69. The variation of current (I) and voltage (V) is as (E)
shown in figure A. The variation of power P with cur- n
O
rent I is best shown by which of the following graph
V 72. In previous problem, if the cell had been con-
nected in parallel (instead of in series) which of the
above graphs would have shown the relationship be-
tween total current I and n ?
Fig A I

394,50 - Rajeev Gandhi Nagar Kota, Ph. No. : 93141-87482, 0744-2209671


IVRS No : 0744-2439051, 52, 53, www. motioniitjee.com , info@motioniitjee.com

Downloaded from www.iitjeephysics4u.com


Page # 54 CURRENT ELECTRICITY

77.When an ammeter of negligible internal resistan ce


is inserted in series with circuit it reads 1A. When the
I/A

I/A
voltmeter of very large resistance is connected across
(A) (B) X it reads 1V. When the point A and B are shorted by
n n a conducting wire, the voltmeter measures 10 V across
O O the battery. The internal resistance of the battery is
equal to

x
I/A

I/A
(C) (D) y
n n
O O
12V
(A) zero (B) 0.5 
(C) 0.2  (D) 0.1 
I/A

(E) 78. Resistances R1 and R2 each 60 are connected in


series as shown in figure. The Potential difference
n
O between A and B is kept 120 volt. Then what will be
the reading of voltmeter connected between the point
73. n identical cells are joined in series with its two C & D if resistance of voltmeter is 120.
A B
cells A and B in the loop with reversed polarities. EMF
of each shell is E and internal resistance r. Potential
difference across cell A or B is (here n > 4)
1 R1 C R2
2E 
(A) (B) 2E 1 –  D
n n
V
(A) 48 V (B) 24 V
4E  2
(C) (D) 2E 1   (C) 40 V (D) None
n n
79. In a galvanometer, the deflection becomes one
half when the galvanometer is shunted by a 20 re-
74. A wire of length L and 3 identical cells of negligible sistor. The galvanometer resistance is
internal resistances are connected is series. Due to
the current, the temperature of the wire is raised by 20
T in time t. N number of similar cells is now con-
nected in series with a wire of the same material and
cross section but of length 2L. The temperature of i/2
the wire is raised by the same amount T in the same
time t. The value of N is : G
(A) 4 (B) 6 (C) 8 (D) 9 i i/2 Rg

75. In the figure shown the power generated in y is (A) 5 (B) 10
maximum when y = 5. Then R is (C) 40 (D) 20
y
80. A galvanometer has a resistance of 20 and reads
full-scale when 0.2 V is applied across it. To convert
10V R it into a 10 A ammeter, the galvanometer coil should
2 have a
(A) 0.01  resistor connected across it
(A) 2  (B) 6  (B) 0.02  resistor connected across it
(C) 5  (D) 3  (C) 200  resistor connected in series with it
(D) 2000  resistor connected in series with it
76. If an ammeter is to be used in place of a voltme-
ter then we must connect with the ammeter a
(A) Low resistance in parallel 81. A milliammeter of range 10mA and resistance 9
(B) High resistance in parallel is joined in a circuit as shown. The metre gives full-
(C) High resistance in series scale deflection for current I when A and B are used
(D) Low resistance in series as its terminals, i.e., current enters at A and leaves
at B (C is left isolated). The value of I is

394,50 - Rajeev Gandhi Nagar Kota, Ph. No. : 93141-87482, 0744-2209671


IVRS No : 0744-2439051, 52, 53, www. motioniitjee.com , info@motioniitjee.com
Downloaded from www.iitjeephysics4u.com
CURRENT ELECTRICITY Page # 55

9 , 10 mA
R

V
(C)
A
A 0.1 0.9 
B C R
(A) 100 mA (B) 900 mA V
(C) 1 A (D) 1.1 A
A
82. A galvanometer coil has a resistance 90 and full (D)
scale deflection current 10 mA. A 910  resistance is
connected in series with the galvanometer to make a
voltmeter. If the least count of the voltmeter is 0.1 V,
the number of divisions on its scale is 86. Shown in the figure below is a meter-bridge set
(A) 90 (B) 91 up with null deflection in the galvanometer. The value
(C) 100 (D) none of the unknown resistor R is
55 R
83. In the circuit shown the resistance of voltmeter
is 10,000 ohm and that of ammeter is 20 ohm. The
ammeter reading is 0.10 Amp and voltmeter reading is
12 volt. Then R is equal to
R C
A

20cm
V
(A) 122  (B) 140  (C) 116  ( D) 100 
84. By error, a student places moving-coil voltmeter
V (nearly ideal) in series with the resistance in a cir-
cuit in order to read the current, as shown. The volt- (A) 220  (B) 110 
meter reading will be (C) 55  (D) 13.75 
87. In a metre bridge experiment, null point is ob-
E=12V, r  2 tained at 20 cm from one end of the wire when resis-
tance X is balnaced against another resistance Y. If X
< Y, then where will be the new position of the null
4 point from the same end, if one decides to balance a
V
resistance of 4X against Y ?
(A) 0 (B) 4V (C) 6V (D) 12V (A) 50 cm (B) 80 cm
(C) 40 cm (D) 70 cm
85. Which of the following wiring diagrams could be 88. In the figure shown for gives values of R1 and R2
used to experimentally determine R using ohm’s law? the balance point for Jockey is at 40 cm from A. When
Assume an ideal voltmeter and an ideal ammeter. R2 is shunted by a resistance of 10 , balance shifts to
R 50 cm. R1 and R2 are (AB = 1m)
R1 R2

V A G
(A)

A B
R
A
10
(A) , 5 (B) 20, 30
(B) V 3
15
(C) 10, 15 (D) 5, 
2

394,50 - Rajeev Gandhi Nagar Kota, Ph. No. : 93141-87482, 0744-2209671


IVRS No : 0744-2439051, 52, 53, www. motioniitjee.com , info@motioniitjee.com

Downloaded from www.iitjeephysics4u.com


Page # 56 CURRENT ELECTRICITY

89. A 6V battery of negligible internal resistance is


internal resistance is 2r. The galvanometer G will show
connected across a uniform wire of length 1m. The
no deflection when the length AJ is
positive terminal of another battery of emf 4V and D ,r
internal resistance 1 is joined to the point A as shown +

in figure. The ammeter shows zero deflection when
the jockey touches the wire at the point C. The AC is J
A B
equal to
6V C
+ –

, 2r G
2
C
A B 4L 5L
(A) (B)
9 9
A 7L 11L
4 V,1 (C) (D)
18 18
(A) 2/3 m (B) 1/3 m
(C) 3/5 m (D) 1/2 m 94. An ammeter A of finite resistance, and a resistor
R are joined in series to an ideal cell C. A potentiom-
90. The figure shows a metre- bridge circuit, with AB eter P is joined in parallel to R. The ammeter reading is
= 100 cm, X = 12 and R = 18, and the jockey J in I0 and the potentiometer reading is V0. P is now re-
the position of balance. If R is now made 8 , through placed by a voltmeter of finite resistance. The amme-
what distance will J have to be moved to obtain ter reading now is I and the voltmeter reading is V.
balance ? C
– +

R
X R A
A J B
P
(A) 10 cm (B) 20 cm (A) I > I0, V < V0 (B) I > I0, V = V0
(C) 30 cm (D) 40 cm (C) I = I0, V < V0 (D) I < I0, V = V0

91. A potentiometer wire has length 10 m and 95. In the given potentiometer circuit length of the
resistance 10. It is connected to a battery of EMF wire AB is 3m and resistance is R = 4.5 . The length
11 volt and internal resistance 1, then the potential AC for no deflection in galvanometer is
gradient in the wire is
(A) 10 V/m (B) 1 V/m E = 5V r  0.5 
(C) 0.1 V/m (D) none
92. The length of a potentiometer wire is l. A cell of R  4.5 
emf E is balanced at a length l/3 from the positive end A B
C
of the wire. if the length of the wire is increased by l/ G
2. At what distance will the same cell give a balance E1 = 3V r1
point.
(A) 2 m (B) 1.8 m
2l l
(A) (B) (C) dependent on r1 (D) none of these
3 2
l 4l
(C) (D)
6 3 96. A battery of emf E0 = 12 V is connected across a
4m long uniform wire having resistance 4/m. The
93. In the figure, the potentiometer wire AB of length cells of small emfs 1 = 2V and 2 = 4V having internal
L and resistance 9r is joined to the cell D of emf  and resistance 2 and 6 respectively, are connected as
internal resistance r. The cell C’s emf is /2 and its shown in the figure. If galvanometer shows no deflec-

394,50 - Rajeev Gandhi Nagar Kota, Ph. No. : 93141-87482, 0744-2209671


IVRS No : 0744-2439051, 52, 53, www. motioniitjee.com , info@motioniitjee.com
Downloaded from www.iitjeephysics4u.com
CURRENT ELECTRICITY Page # 57

tion at the point N, the distance of point N from the STATEMENT-2 is a correct explanation for STATEMENT-
point A is equal to 1
0 R  8  (B) STATEMENT-1 is True, STATEMENT-2 is True;
STATEMENT-2 is NOT a correct explanation for STATE-
MENT-1
N (C) STATEMENT-1 is True, STATEMENT-2 is False
A
B (D) STATEMENT-1 is False, STATEMENT-2 is True
1 r1 G 100. Statement-1 : Knowing that rating is done at
steady state of the filament, an electric bulb con-
2 r2 nected to a source having rated voltage consumes
1 1 more than rated power just after it is switched on.
(A) m (B) m
6 3 Statement-2 : When filament is at room temperature
(C) 25 cm (D) 50 cm
its resistance is less than its resistance when the
bulb is fully illuminated
97. In the arrangement shown in figure when the
(A) STATEMENT-1 is True, STATEMENT-2 is True;
switch S2 is open, the galvanometer shows no deflec-
STATEMENT-2 is a correct explanation for STATEMENT-
tion for l = L/2. When the switch S2 is closed, the 1
galvanometer shows no deflection for l = 5L/12. The (B) STATEMENT-1 is True, STATEMENT-2 is True;
internal resistance (r) of 6 V cell, and the emf E of the STATEMENT-2 is NOT a correct explanation for STATE-
other battery are respectively.
10  MENT-1
( )

S2 (C) STATEMENT-1 is True, STATEMENT-2 is False


6V r (D) STATEMENT-1 is False, STATEMENT-2 is True
G
l
A B 101. Statement-1 : When a battery is supplying
L power to a circuit, work done by electrostatic forces
( )
E S1 on electrolyte ions inside the battery is positive
Statement-2 : Electric field is directed from positive
(A) 3, 8V (B) 2, 12V
to negative electrode inside a battery
(C) 2, 24V (D) 3, 12V
(A) STATEMENT-1 is True, STATEMENT-2 is True;
STATEMENT-2 is a correct explanation for STATEMENT-
98. Statement-1 : When two conducting wires of dif-
1
ferent resistivity having same cross section area are
(B) STATEMENT-1 is True, STATEMENT-2 is True;
joined in series, the electric field in them would be
STATEMENT-2 is NOT a correct explanation for STATE-
equal when they carry current.
MENT-1
Statement-2 : When wires are in series they carry
(C) STATEMENT-1 is True, STATEMENT-2 is False
equal current.
(D) STATEMENT-1 is False, STATEMENT-2 is True
(A) STATEMENT-1 is True, STATEMENT-2 is True;
STATEMENT-2 is a correct explanation for STATEMENT-
102. Statement-1 : Conductivity of a metallic con-
1
ductor decreases with increase in temperature.
(B) STATEMENT-1 is True, STATEMENT-2 is True;
Statement-2 : On increasing temperature the number
STATEMENT-2 is NOT a correct explanation for STATE-
of free electrons in the metallic conductar decreases.
MENT-1
(A) STATEMENT-1 is True, STATEMENT-2 is True;
(C) STATEMENT-1 is True, STATEMENT-2 is False
STATEMENT-2 is a correct explanation for STATEMENT-
(D) STATEMENT-1 is False, STATEMENT-2 is True
1
(B) STATEMENT-1 is True, STATEMENT-2 is True;
99. Statement-1 : Potential difference across the ter-
STATEMENT-2 is NOT a correct explanation for STATE-
minals of a battery is always less than its emf.
MENT-1
Statement-2 : A battery always has some internal
(C) STATEMENT-1 is True, STATEMENT-2 is False
resistance.
(D) STATEMENT-1 is False, STATEMENT-2 is True
(A) STATEMENT-1 is True, STATEMENT-2 is True;

394,50 - Rajeev Gandhi Nagar Kota, Ph. No. : 93141-87482, 0744-2209671


IVRS No : 0744-2439051, 52, 53, www. motioniitjee.com , info@motioniitjee.com

Downloaded from www.iitjeephysics4u.com


Page # 58 CURRENT ELECTRICITY

Exercise - II (MULTIPLE
(Multiple CORRECT
Correct PROBLEMS)
Problems)
1. A battery is of emf E is being charged from a charger (A) the charge crossing in a given time interval.
such that positive terminal of the battery is connected (B) drift speed
to terminal A of charger and negative terminal of the (C) current density
battery is connected to terminal B of charger. The (D) free-electron density
internal resistance of the battery is r.
6. A simple circuit contains an ideal battery and a
(A) Potential difference across points A and B must
resistance R. If a second resistor is placed in parallel
be more than E
with the first.
(B) A must be at higher potential than B
(A) the potential across R will decrease
(C) In battery, current flows from positive terminal to
(B) the current through R will decreased
the negative terminal
(C) the current delivered by the battery will increase
(D) No current flows through battery
(D) the power dissipated by R will increased.
2. A battery of emf E and internal resistance r is con-
7. In the circuit shown E, F, G and H are cells of e.m.f.
nected across a resistance R. Resistance R can be
2V, 1V, 3V and 1V respectively and their internal re-
adjusted to any value greater than or equal to zero. A
sistances are 2, 1, 3 and 1 respectively.
graph is plotted between the current (i) passing
through the resistance and potential differ ence (V) A
across it. Select the F E
correct alternative (s).
V D B
(volt)
2
10
G H
i
C
(A) VD – VB = –2/13 V
2 ampere
(B) VD – VB = 2/13 V
(C) VG = 21/13 V = potential difference across G.
(A) internal resistance of battery is 5
(D) VH = 19/13 V = potential difference across H.
(B) emf of the battery is 20V
(C) maximum current which can be taken from the 8. Consider the circuit shown in the figure
battery is 4A 5 3  A 3
(D) V-i graph can never be a straight line as shown in
figure.
28 V 10  10  4
3. Which of the following quantities do not change
when a resistor connected to a battery is heated due
to the current? 4 B 2 3
(A) drift speed (B) resistivity (A) the current in the 5  resistor is 2A
(C) resistance (D) number of free electrons. (B) the current in the 5 resistor is 1A
(C) the potential difference VA – VB is 10 V
4. A metallic conductor of irregular cross-section is as (D) the potential difference VA – VB is 5V
shown in the figure. A constant potential difference is
9. The equivalent resistance of a group of resistances
applied across the ends (1) and (2). Then :
is R. If another resistance is connected in parallel to
the group, its new equivalent becomes R1 & if it is
(1) ×P ×Q (2) connected in series to the group, its new equivalent
becomes R2 we have
(A) the current at the cross-section P equals the (A) R1 > R (B) R1 < R
current at the cross-section Q (C) R2 > R (D) R2 < R
(B) the electric field intensity at P is less than that at 10. The value of the resistance R in figure is adjusted
Q. such that power dissipated in the 2 resistor is
(C) the rate of heat generated per unit time at Q is maximum. Under this condition
greater than that at P
8
(D) the number of electrons crossing per unit area of
cross-section at P is less than that at Q.
12V
5. A current passes through a wire of nonuniform cross R 2
section. Which of the following quantities are inde-
pendent of the cross-section?

394,50 - Rajeev Gandhi Nagar Kota, Ph. No. : 93141-87482, 0744-2209671


IVRS No : 0744-2439051, 52, 53, www. motioniitjee.com , info@motioniitjee.com
Downloaded from www.iitjeephysics4u.com
CURRENT ELECTRICITY Page # 59

(A) R = 0
(B) R = 8 (A) If the devices are ideal, ammeter will read zero
(C) power dissipated in the 2 resistor is 72 W current and voltmeter will read the emf of cell.
(D) power dissipated in the 2 resistor is 8 W (B) If the devices are ideal, a large current will flow
through the ammeter and it will be damaged.
11. A bulb is connected to a battery of emf 10 V so (C) The main current in the circuit will be very low
that the resulting current is 10 mA. When the bulb is and practically all current will flow through the amme-
connected to 220 V mains, the current is 50 mA. ter, if resistance of ammeter is much smaller than the
Choose the correct alternative (s) resistance in parallel.
(A) In the first case, the resistance of the bulb is 1k (D) The devices may get damaged if emf of the cell is
and in second case, it is 4.4 k very high and the meters and nonideal.
(B) It is not possible since ohm’s law is not followed
(C) the increase in resistance is due to heating of the 17. In the given potentiometer circuit, the resistance
filament of the bulb when it is connected to 220 V of the potentiometer wire AB is R0. C is a cell of inter-
mains nal resistance r. The galvanometer G does not give
(D) None of these zero deflection for any position of the Jockey J. Which
of the following cannot be a reason for this?
12. A galvanometer may be converted into ammeter D R
+ –
or voltmeter. In which of the following cases the re-
sistance of the device will be the largest ? (Assume A B
maximum range of galvanometer = 1mA) r J
C
(A) an ammeter of range 10A
G
(B) a voltmeter of range 5 V
(A) r > R0 (B) R >> R0
(C) an ammeter of range 5A
(C) emf of C > emf of D
(D) a voltmeter of range 10 V.
(D) The negative terminal of C is connected A.
13. Mark out the correct options.
18. In a potentiometer arrangement. E1 is the cell
(A) An ammeter should have small resistance.
establishing current in primary circuit. E2 is the cell to
(B) An ammeter should have large resistance.
be measured. AB is the potentiometer wire and G is a
(C) A voltmeter should have small resistance.
galvanometer. Which of the following are the essen-
(D) A voltmeter should have large resistance.
tial condition for balance to be obtained.
(A) The emf of E1 must be greater than the emf of E2
14. In the circuit shown the readings of ammeter and
(B) Either the positive terminals of both E1 and E2 or
voltmeter are 4A and 20V respectively. The meters
the negative terminals of both E1 and E2 must be joined
are non ideal, then R is :
to one end of potentiometer wire.
R (C) The positive terminals of E1 and E2 must be joined
A
to one end of potentiometer wire.
V (D) The resistance of G must be less than the resis-
(A) 5 (B) less than 5 tance of AB.
(C) greater than 5 (D) between 4 & 5 19. In a potentiometer wire experiment the emf of a
15. A micrometer has a resistance of 100 and a full battery in the primary circuit is 20V and its internal
scale range of 50A. It can be used as a voltmeter or resistance is 5. There is a resistance box in series
a higher range ammeter provided a resistance is added with the battery and the potentiometer wire, whose
to it. Pick the correct range and resistance resistance can be varied from 120  to 170 . Resis-
combination(s). tance of the potentiometer wire is 75. The following
(A) 50 V range with 10 kresistance in series potential differences can be measured using this po-
(B) 10 V range with 200 k resistance in series tentiometer.
(C) 5 mA range with 1  resistance in parallel (A) 5V (B) 6V
(D) 10 mA range with 1 k resistance in parallel. (C) 7V (D) 8V
20. Two indentical fuses are rated at 10A. If they are
16. By mistake, a voltmeter is placed in series and an joined
ammeter is parallel with a resistance in an electric (A) in parallel, the combination acts as a fuse of rat-
circuit, with a cell in series. ing 20A
(B) in parallel, the combination acts as a fuse of rat-
ing 5A
(C) in series, the combination acts as a fuse of rating
10A
(D) in series, the combination acts as a fuse of rating
20A

394,50 - Rajeev Gandhi Nagar Kota, Ph. No. : 93141-87482, 0744-2209671


IVRS No : 0744-2439051, 52, 53, www. motioniitjee.com , info@motioniitjee.com

Downloaded from www.iitjeephysics4u.com


Page # 60 CURRENT ELECTRICITY

Exercise - III (Subjective Problems)


1. A current of 0.50 ampere is passing through a CuSO4
solution. How many Cu++ ions will be deposited on 2
X + –
cathode in 10 seconds ?
+ 4V
2. A copper wire of radius 0.1 mm and resistance 1 k – 2V
3 – 3 5
is connected across a power supply of 20 V. (a) How + 4V
many electrons are transferred per second between Y
the supply and the wire at one end ? (b) Write down
the current density in the wire. (b) If intermediate cell has internal resistance r = 1 
then determine the potential difference between X and Y.
3. A wire has a length of 2.0 m and a resistance of 8. For what value of R in circuit, current through 4
5.0 . Find the electric field existing inside the wire if resistance is zero.
it carries a current of 10 A.
R
4. (a) A car has a fresh storage battery of emf 12 V
and internal resistance 5.0 × 10–2 . If the starter
draws a current of 90 A, what is the terminal volt- 4V 6V
age of the battery when the starter is on ? 10V
(b) After long use, the internal resistance of the stor-
age battery increases to 500 . What maximum cur- 9. In the circuit shown in figure the reading of amme-
rent can be drawn from the battery ? Assume the emf ter is the same with both switches open as with both
of the battery to remains unchanged. 0closed. Then find the resistance R. (ammeter is ideal)
(c) If the discharged battery is charged by an exter- 100
nal emf source, is the terminal voltage of the battery A
during charging greater of less than its emf 12 V ?
R 50 
5. Find the current through the 10  resistor shown in
figure.
10  3 V
+ –
300  1.5 V

3 6
10. If the switches S1, S2 and S3 in the figure are
arranged such that current through the battery is
4.5 V minimum, find the voltage across points A and B.
6  3
6. For the circuit shown in figure, determine the un-
known voltage drop V1. 6 A
S2

+2V – +1V – s1
1 9 1 1
S3
+ + 24 V
V1
30 V I B
– –
11. The resistance of the rheostat shown in figure is
30 . Neglecting the meter resistance, find the minimum
and maximum currents through the ammeter as the
– 5V + – 3V + rheostat is varied.5.5 V
A
7. (a) Determine the potential difference between X
and Y in the circuit shown in figure. 10 

30 
20 

394,50 - Rajeev Gandhi Nagar Kota, Ph. No. : 93141-87482, 0744-2209671


IVRS No : 0744-2439051, 52, 53, www. motioniitjee.com , info@motioniitjee.com
Downloaded from www.iitjeephysics4u.com
CURRENT ELECTRICITY Page # 61

12. Find the current I & voltage V in the circuit shown. 17. In the circuit shown in figure, all wires have equal
resistance r. Find the equivalent resistance between
5  7 A and B. C
60 V E

41 7  B
D
F
I V 0.4 
20 V 4  A
8
18. In given circuit determine.
2 10 (a) The rate at which the chemical energy of the cell
is consumed
13. For the circuit shown in the figure, find the volt- (b) The rate at which heat is generated inside the
age across 10  resistor and the current passing battery
through it. (c) Electric power output
(d) Which resistance consumes maximum power
(e) Power dissipated across 4 resistance
5 10  10A 2 1 5A
i1 8
current source current source
i2 4
14. In the circuit, the galvanometer G shows zero i3 8
deflection. If the batteries A and B have negligible
internal resitance, the value of the resistor R will be : i

500
G 2V E = 6V, r  1
A 19. Three equal resistors connected in series across
R a source of emf together dissipate 10 watts of power.
What would be the power dissipated if the same re-
12V B
sistors are connected in parallel across the same
source of emf ?
(A) 200  (B) 100 
20. Find the current through 25V cell & power supplied
(C) 500  (D) 1000 
by 20V cell in the figure shown.
15. A network of nine conductors connects six points
A, B, C, D, E and F as shown in figure. The figure 10V 5V 20V 30V
denotes resistances in ohms. Find the equivalent 25V
A
resistance between A and D.
11
10 
5

5

2
1 1 D
1
1 21. If a cell of constant E.M.F. produces the same
amount of the heat during the same time in two in
E F dependent resistors R1 and R2, when they are sepa-
2 1 2 rately connected across the terminals of the cell, one
B C after the another, find the internal resistance of the
1 cell.
16. Find the equivalent resistance of the circuit be- 22. One kilowatt electric heater is to be used with
tween points A and B shown in figure is : (each branch 220 V.D.C supply.
is of resistance = 1) (a) What is the current in the heater
(b) What is its resistance.
(c) What is the power dissipated in the heater.
A B (d) How much heat in calories is produced per sec-
ond.
(e) How many grams of water at 100ºC will be con-
verted per minute into steam at 100ºC with the heater.
[(latent heat of vaporisation of water = 540 cal/g)]

394,50 - Rajeev Gandhi Nagar Kota, Ph. No. : 93141-87482, 0744-2209671


IVRS No : 0744-2439051, 52, 53, www. motioniitjee.com , info@motioniitjee.com

Downloaded from www.iitjeephysics4u.com


Page # 62 CURRENT ELECTRICITY

23. The efficiency of a cell when connected to a


resistance R is 60%. What will be its efficiency if the 28. A battery of emf 0 = 10 V is connected across a
external resistance is increased to six times. 1m long uniform wire having resistance 10/m. Two
cells of emf 1 = 2V and 2 = 4V having internal resis-
24. In following diagram boxes may contain resistor or tances 1 and 5 respectively are connected as shown
battery or any other element in the figure. If a galvanometer shows no deflection
at the point P, find the distance of point P from the
point a.
1A=i 10

 0  10 V
10 V,1 P
(A)
A B
1

 1  2V G
1A= i 5

 2  4V
10 V,1 29. A potentiometer wire AB is 100 cm long and has a
(B) total resistance of 10ohm. If the galvanometer shows
then determine in each case zero deflection at the position C, then find the value
(A) E.m.f of battery of unknown resistance R :
(B) Battery is acting as a source or load
(C) Potential difference across each battery. 10 V
(D) Power input to the battery or output by the bat-
40 cm
tery. C
(E) The rate at which heat is generated inside the A B
5V
battery. G
(F) The rate at which the chemical energy of the cell r  1
is consumed or increased.
(G) Potential difference across box. R
(H) Electric power output across box. 30. In the figure shown for which values of R1 and R2
25. Find the resistor in which maximum heat will be the balance point for Jockey is at 40 cm from A. When
produced. R2 is shunted by a resistance of 10, balance shifts to
5 5 50 cm. Find R1 and R2. (AB = 1m)
R1 R2
6
4 G
2
A B
V
26. A part of a circuit is shown in figure. Here reading
31. An accumulator of emf 2 volt and negligible inter-
of ammeter is 5 ampere and voltmeter is 96V & volt-
nal resistance is connected across a uniform wire of
meter resistance is 480 ohm. Then find the resistance R
R length 10m and resistance 30. The appropriate ter-
A minals of a cell of emf 1.5 Volt and internal resistance
1 is connected to one end of the wire, and the other
terminal of the cell is connected through a sensitive
V galvanometer to a slider on the wire. What length of
the wire will be required to produce zero deflection of
27. The ammeter shown in figure consists of a 480  the galvanometer ? How will the balancing change (a)
coil connected in parallel to a 20  shunt. Find the when a coil of resistance 5 is placed in series with
reading of the ammeter. the accumulator, (b) the cell of 1.5 volt is shunted
A with 5 resistor ?
10  Sol.

20 V

394,50 - Rajeev Gandhi Nagar Kota, Ph. No. : 93141-87482, 0744-2209671


IVRS No : 0744-2439051, 52, 53, www. motioniitjee.com , info@motioniitjee.com
Downloaded from www.iitjeephysics4u.com
CURRENT ELECTRICITY Page # 63

Exercise - IV (Tough Subjective Problems)


1. A triangle is constructed using the wires AB, BC & 6. A network of resistance is constructed with R1 & R2
CA of same material and of resistance , 2 & 3 as shown in the figure. The potential at the points
respectively. Another wire of resistance /3 from A 1,2,3,.., N and V1, V2, V3,....., Vn respectively each
can make a sliding contact with wire BC. Find the having a potential k time smaller than previous one.
maximum resistance of the network between points A R1 R2
and the point of sliding wire with BC. Find (i) R and R in terms of k
2 3
2. (a) The current density across a cylindrical con-
V0 R1 V1 R1 V2 R1 V3 VN–1 R1 VN
ductor of radius R varies according to the equation

 r
J  J0  1   , where r is the distance from the axis.
 R V0 = kV1 R2 R3
R2 R2 R2
Thus the current density is a maximum J0 at the axis r
= 0 and decreases linearly to zero at the surface r =
R. Calculate the current in terms of J0 and the
(ii) current that passes through the resistance R2 near-
conductor’s cross sectional area is A = R2.
est to the V0 in terms V0, k & R3.
(b) Suppose that instead the current density is a
maximum J0 at the surface and decreases linearly to 7. A person decides to use his bath tub water to
generate electric power to run a 40 watt bulb. The
r
zero at the axis so that J  J0 . Calculate the cur- bath tube is located at a height of 10m from the
R ground & it holds 200 litres of water. If we install a
rent. water driven wheel generator on the ground, at what
3. What will be the change in the resistance of a rate should the water drain from the bath tube to
circuit consisting of five identical conductors if two light bulb ? How long can we keep the bulb on, if the
similar conductors are added as shown by the dashed bath tub was full initially. The efficiency of generator
line in figure is 90%.(g = 10m/s–2)
8. In the circuit shown in figure, calculate the follow-
ing
(i) Potential difference between points a and b when
switch S is open. (ii) Current through S in
the circuit when S is closed.
4. The current I through a rod of a certain metallic
36v
oxide is given by I = 0.2 V5/2, where V is the potential
difference across it. The rod is connected in series
with a resistance to a 6V battery of negligible internal
resistance. What value should the series resistance a S b
have so that
(i) the current in the circuit is 0.44
(ii) the power dissipated in the rod is twice that dissi-
pated in the resistance.

5. A piece of resistive wire is made up into two squares 9. The circuit shown in figure is made of a homoge-
with a common side of length 10 cm. A current enters neous wire of uniform cross-section. ABCD is a square.
the rectangular system at one of the corners and
leaves at the diagonally opposite corners. Show that Find the ratio of the amounts of heat liberated per
the current in the common side is 1/5th of the enter- unit time in wire A-B and C-D.
ing current. What length of wire connected between D C
input and output terminals would have an equivalent
effect.
A B

394,50 - Rajeev Gandhi Nagar Kota, Ph. No. : 93141-87482, 0744-2209671


IVRS No : 0744-2439051, 52, 53, www. motioniitjee.com , info@motioniitjee.com

Downloaded from www.iitjeephysics4u.com


Page # 64 CURRENT ELECTRICITY

10. A rod of length L and cross-section area A lies (a) Find the length AP of the wire such that the gal-
along the x-axis between x = 0 and x = L. The mate- vanometer shows zero deflection.
rial obeys Ohm’s law and its resistivity varies along (b) Now the rheostat is put at maximum resistance
the rod according to (x) = 0e–x/L. The end of the rod (10 ) and the switch S is closed. New balancing
at x = 0 is at a potential V0 and it is zero at x = L. length is found to 8m. Find the internal resistance r of
(a) Find the total resistance of the rod and the cur- the 4.5 V cell.
rent in the wire. 10V
(b) Find the electric potential in the rod as a function
of x. P
A B
11. An ideal cell having a steady emf of 2 volt is 12m
G
connected across the potentiometer wire of length
10 m. The potentiometer wire is of magnesium and 4.5V r S
having resistance of 11.5 /m. An another cell gives
a null point at 6.9 m. If a resistance of 5 is put in
series with potentiometer wire, find the new position 17. A galvanometer (coil resistance 99) is converted
of the null point. into a ammeter using a shunt of 1 and connected as
12. An enquiring physics student connects a cell to a shown in the figure (i). The ammeter reads 3A. The
circuit and measures the current drawn from the cell same galvanometer is converted into a voltmeter by
to I1. When he joins a second identical cell is series connecting a resistance of 101  in series. This
with the first, the current becomes I2. When the cells voltmeter is connected as shown in figure (ii). Its
are connected are in parallel, the current through the reading is found to be 4/5 of the full scale reading.
circuit is I3. Show that relation between the current is Find
3 I3 I2 = 2 I1 (I2 + I3)
13. Find the potential difference VA – VB for the cir- 12V r 12V r
cuit shown in the figure.
1V 1V B 1V 1V A
IV IV V
(i) (ii)
A
1V 1V 1V 1V
(a) internal resistance r of the cell
14. A resistance R of thermal coefficient of resistivity (b) range of the ammeter and voltmeter
=  is connected in parallel with a resistance = 3R, (c) full scale deflection current of the galvanometer
having thermal coefficient of resistivity = 2. Find the
value of eff.
15. A galvanometer having 50 divisions provided with
a variable shunt s is used to measure the current
when connected in series with a resistance of 90 
and a battery of internal resistance 10 . It is ob-
served that when the shunt resistance are 10, 50,
respectively the deflection are respectively 9 & 30
divisions. What is the resistance of the galvanometer
? Further if the full scale deflection of the galvanom-
eter movement is 300 mA, find the emf of the cell.

16. In the primary circuit of potentiometer the rheo-


stat can be varied from 0 to 10. Initially it is at
minimum resistance (zero)

394,50 - Rajeev Gandhi Nagar Kota, Ph. No. : 93141-87482, 0744-2209671


IVRS No : 0744-2439051, 52, 53, www. motioniitjee.com , info@motioniitjee.com
Downloaded from www.iitjeephysics4u.com
CURRENT ELECTRICITY Page # 65

Exercise - V JEE-Problems
1. An electrical circuit is shown in the figure. Calcu- (A) IR = IG (B) IP = IG
late the potential difference across the resistance of (C) IQ = IG (D) IQ = IR
400 ohm, as will be measured by the voltmeter V of
5. The effective resistance between the points P and
resistance 400 ohm, either by applying Kirchhoff’s rules
Q of the electrical circuit shown in the figure is
or otherwise. [JEE-96]
2R 2R
V 2R

P r r Q [JEE 2002]
2R
2R 2R

(A) 2 Rr / (R + r) (B) 8R(R + r)/(3R + r)


(C) 2r + 4R (D) 5R/2 + 2r
10V

2. (i) A steady current flows in a metallic conductor 6. A 100 W bulb B1, and two 60 W bulbs B2 and B3, are
of nonuniform cross-section. The quantity/quantities connected to a 250 V source, as shown in the figure.
constant along the length of the conductor is / are : Now W1, W2 and W3 are the output powers of the
[JEE,97] bulbs B1, B2 and B3 respectively. Then
(A) current, electric field and drift speed
(B) drift speed only B1 B2
(C) current and drift speed
(D) current only
(ii) The dimension of electricity conductivity is
............. .
(iii) Find the emf (E) & internal resistance (r) of a B3
single battery which is equivalent to a parallel combi-
nation of two batteries of emfs V1 & V2 & internal
resistances r1 & r2 respectively with their similar po-
larity connected to each other
(A) W1 > W2 = W3 250 VW >W >W
(B) 1 2 3
3. In the circuit shown in the figure, the current
(C) W1 < W2 = W3 (D) W1 < W2 < W3
through
7. A thin uniform wire AB of length 1m, an unknown
resistance X and a resistance of 12 are connected
by thick conducting strips, as shown in figure. A bat-
tery and a galvanometer (with a sliding jockey con-
9V nected to it) are also available. Connections are to be
made to measure the unknown resistance X using the
principle of Wheatstone bridge. Answer the following
question.
(A) the 3 resistor is 0.50 A
(B) the 3 resistor is 0.25 A
(C) 4 resistor is 0.50 A x 12 
(D) the 4 resistor is 0.25 A [JEE’98]
A B C D
4. In the circuit shown, P  R, the reading of the (A) Are there positive and negative terminals on the
galvanometer is same with switch S open or closed. galvanometer ?
Then [JEE’ 99] (B) Copy the figure in your answer book and show the
battery and the galvanometer (with jockey) connected
P Q at appropriate points.
(C) After appropriate connections are made, it is found
R S
G
that no deflection takes place in the galvanometer
when the sliding jockey touches the wire at a dis-
tance of 60 cm from A. Obtain the value of resistance X.
V
[JEE-2002]

394,50 - Rajeev Gandhi Nagar Kota, Ph. No. : 93141-87482, 0744-2209671


IVRS No : 0744-2439051, 52, 53, www. motioniitjee.com , info@motioniitjee.com

Downloaded from www.iitjeephysics4u.com


Page # 66 CURRENT ELECTRICITY

8. Arrange the order of power dissipated in the given P


circuits, if the same current is passing through all
circuit and each resistor is ‘r’

(I) A B

(ii) A Q R
B
[JEE-2004]
(A) P and Q (B) Q and R
(iii) A B (C) P and R (D) any two points

12. For the post office box arrangement to determine


the value of unknown resistance, the unknown resis-
(iv) A tance should be connected between
B
[JEE-2004]
B C D
(A) P2 > P3 > P4 > P1 (B) P3 > P2 > P4 > P1
(C) P4 > P3 > P2 > P1 (D) P1 > P2 > P3 > P4

9. In the given circuit, no current is passing through A


the galvanometer. If the cross-sectional diameter of
AB is doubled the for null point of galvanometer the B1 C1
value of AC would (A) B and C (B) C and D
(C) A and D (D) B1 and C1

13. Draw the circuit for experimental verification of


G Ohm’s law using a source of variable D.C. voltage, a
A main resistance of 100, two galvanometers and two
B
C resistances of values 106  and 10–3  respectively.
x
(A) x (B) x/2 Clearly show the positions of the voltmeter and the
(C) 2x (D) none ammeter.
[JEE-2003] [JEE-2004]

10. How a battery is to be connected so that shown 14. In the figure shown the current through 2 resis-
rheostat will behave like a potential divider ? Also tor is
indicate the points about which output can be taken. 10 
10 

[JEE-2003]
C 20 
5
2
A B
R (A) 2A (B) 0A
(C) 4A (D) 6A
11. Six equal resistances are connected between points
[JEE - 2004]
P, Q and R as shown in the figure. Then the net resis-
tance will be maximum between 15. A galvanometer has resistance 100 and it re-
quires current 100A for full scale deflection. A resis-
tor 0.1 is connected to make it an ammeter. The

394,50 - Rajeev Gandhi Nagar Kota, Ph. No. : 93141-87482, 0744-2209671


IVRS No : 0744-2439051, 52, 53, www. motioniitjee.com , info@motioniitjee.com
Downloaded from www.iitjeephysics4u.com
CURRENT ELECTRICITY Page # 67

smallest current required in the circuit to produce the 19. Figure shows three resistor configurations R1, R2
full scale deflection is and R3 connected to 3V battery. If the power dissi-
[JEE-2005] pated by the configuration R1, R2 and R3 is P1, P2
(A) 1000.1 mA (B) 1.1 mA and P3, respectively, then
(C) 10.1 mA (D) 100.1 mA

16. An unknown resistance X is to be determined us- 3V


3V
ing resistances R1, R2 or R3. Their corresponding null 3V
points are A, B and C. Find which of the above will
give the most accurate reading and why ? R2
R1 R3

[JEE - 2008]
(A) P1 > P2 > P3 (B) P1 > P3 > P2
(C) P2 > P1 > P3 (D) P3 > P2 > P1

X R 20. STATEMENT-1
In a Meter Bridge experiment, null point for an un-
known resistance is measured. Now, the unknown re-
G sistance is put inside an enclosure maintained at a
higher temperature. The null point can be obtained at
the same point as before by decreasing the value of
A B C the standard resistance.
R = R1 or R2 or R3 and
[JEE-2005]
STATEMENT-2
Resistance of a metal increases with increase in tem-
17. Consider a cylindrical element as shown in the perature.
figure. Current flowing the through element is I and (A) STATEMENT-1 is True, STATEMENT-2 is True;
STATEMENT-2 is a correct explanation for STATEMENT-
resistivity of material of the cylinder is . Choose the
1
correct option out the following [JEE-2006] (B) STATEMENT-1 is True, STATEMENT-2 is True;
A B STATEMENT-2 is NOT a correct explanation for STATE-
I C MENT-1
4r 2r (C) STATEMENT-1 is True, STATEMENT-2 is False
(D) STATEMENT-1 is False, STATEMENT-2 is True
l/2 l/2 [JEE - 2008]
(A) Power loss in second half is four times the power
loss in first half 21. For the circuit shown in the figure
(B) Voltage drop in first half is twice of voltage drop in
second half
(C) Current density in both halves are equal
(D) Electric field in both halves is equal

18. A resistance of 2 is connected across one gap


(A) the current I through the battery is 7.5 mA
of a metre-bridge (the length of the wire is 100 cm)
(B) the potential difference across RL is 18 V
and an unknown resistance, greater than 2, is con- (C) ratio of powers dissipated in R1 and R2 is 3
nected across the other gap. When these resistances (D) if R1 and R2 are interchanged, magni tude of the
are interchanged, the balance point shifts by 20 cm. power dissipated in RL will de crease by a factor of 9
[JEE - 2009]
Neglecting any corrections, the unknown resistance is
[JEE-2007] 22. Consider a thin square sheet of side L and thick-
(A) 3  (B) 4  ness t, made of a material of resistivity . The resis-
(C) 5  (D) 6  tance between two opposite faces shown by the

394,50 - Rajeev Gandhi Nagar Kota, Ph. No. : 93141-87482, 0744-2209671


IVRS No : 0744-2439051, 52, 53, www. motioniitjee.com , info@motioniitjee.com

Downloaded from www.iitjeephysics4u.com


Page # 68 CURRENT ELECTRICITY

26. A meter bridge is set-up as shown, to determine


shaded area in the figure is : [JEE 2010]
an unknown resistance 'X' using a standard 10 ohm
resistor. The galvanometer shows null point when tap-
ping-key is at 52 cm mark. The end-corrections are 1
cm and 2 cm respectively for the ends A and B. The
determined value of 'X' is [JEE 2011]
t
L
X 10
(A) directly proportional to L
(B) directly proportional to t
(C) independent of L
(D) independent of t

23. Incandenscent bulbs are designed by keeping in


mind that the resitance of their filament increases A B
with the increase in temperature. If at room tempera-
ture, 100 W, 60 W and 40 W bulbs have filament re- (A) 10.2 ohm (B) 10.6 ohm
sistances R100, R60 and R40, respectively, the relation (C) 10.8 ohm (D) 11.1 ohm
between these resistance is [JEE 2010]
1 1 1 27. Two batteries of different emfs and different in-
(A) R   (B) R100 = R40 + R60 ternal resistances are connected as shown. The volt-
100 R 40 R 60
age across AB in volts is [JEE 2011]
1 1 1 6V
(C) R100 > R60 > R40 (D) R  
100 R 60 R 40

A B
24. To verify Ohm's law, a student is provided with a
test resistor RT. a high resistance R1, a small resis-
tance R2, two identical galvanometers G1 and G2 and a
variable voltage source V. the correct circuit to carry
out the experiment is :[JEE 2010] 3V network 2
28. For the resistance shown in the figure,
choose the correct option(s).
G1 G1 I2 

R2 R1 
G2 G2  
RT R1 RT R2
(A) (B) 


v v I1

(A) The current through PQ is zero


R1 R2
G1 G1 (B) l1 = 3A
(C) The potential at S is less than that at Q
G2 G2
RT RT
(D) l2 = 2A [JEE 2012]
(C) R2
(D) R1

v v

25. When two identical batteries of internal resistance


1  each are connected in series across a resistor R,
the rate of heat produced in R is J1. When the same
batteries are connected in parallel across R, the rate
is J2. If J1 = 2.25 J2 then the value of R in  is
[JEE 2010]

394,50 - Rajeev Gandhi Nagar Kota, Ph. No. : 93141-87482, 0744-2209671


IVRS No : 0744-2439051, 52, 53, www. motioniitjee.com , info@motioniitjee.com
Downloaded from www.iitjeephysics4u.com
CURRENT ELECTRICITY Page # 69

Exercise-I

1. B 2. B 3. D 4. A 5. A 6. C 7. C
8. C 9. D 10. D 11. D 12. D 13. B 14. B

15. D 16. C 17. A 18. C 19. D 20. A 21. A


22. A 23. B 24. B 25. D 26. D 27. C 28. B
29. A 30. B 31. A 32. B 33. C 34. C 35. D
36. D 37. B 38. C 39. B 40. A 41. B 42. C
43. B 44. D 45. C 46. C 47. B 48. D 49. D
50. B 51. B 52. B 53 A 54. B 55. B 56. C
57. B 58. A 59. C 60. A 61. B 62. D 63. A
64. A 65. D 66. B 67. A 68. A 69. B 70. B
71. D 72. A 73. D 74. B 75. D 76. C 77. C
78. A 79. D 80. B 81. C 82. C 83. D 84. D
85. B 86. A 87. A 88. A 89. A 90. B 91. B

92. B 93. B 94. A 95. D 96. C 97. B 98. D

99. D 100. A 101. D 102. C

Exercise-II

1. A,B,C 2. A 3. D 4. A,B,C,D 5. A,D 6. C 7. A,C,D


8. A 9. C 10. A,C 11. A,C 12. D 13. A,D 14. C
15. B,C 16. A,C,D 17. A 18. A,B 19. A,B,C 20. A,C

Exercise-III
2
1. 1.5625 × 1019 2. 1.25 × 10–17, (b) × 106 A/m2 3. 25 V/m 4. (a) 7.5 V, (b) 24 mA(c) greater than 12 V

5. zero 6. 19 V 7. (a) 3.7 V (b) 3.7 V 8. 1 9. 600

3 3  5 .5
10. 1V 11.  0.15 A ,  0.83 A 12. I = 2.5 A, V = 3.5 volts
20 20

25 5 22 3r
13. V  2.78 V, A  0.278 A 14. B 15. 1W 16.  17.
9 18 35 5

18. (a) 12 W (b) 4W (c) 8 W (d) 4 (e) 4W

19. 90 watt. 20. 12A, – 20 W 21. R1 R 2

394,50 - Rajeev Gandhi Nagar Kota, Ph. No. : 93141-87482, 0744-2209671


IVRS No : 0744-2439051, 52, 53, www. motioniitjee.com , info@motioniitjee.com

Downloaded from www.iitjeephysics4u.com


Page # 70 CURRENT ELECTRICITY

50 22  11
22. (a)  4.55 A (b)  48 .4  (c) 1000 W (d) 240 cal s–1 (e) 80/3 gm
11 5

23. 90%
24. (a) E = 10 V each (b) (A) act as a source and (B) act as load
(c) VA = 9V, VB = 11 V (d) PA = 9 W, PB = 11 W (e) Heat rate = 1 W each
(f) 10 watt. (g) 9V, 11 V (b) –9W, 11 W
50
25. 4 26. 20 ohm 27. A 28. 46.67 cm
73
10
29. 4 ohm 30. , 5 31. 7.5 m, 8.75 m, 6.25 m
3

Exercise-IV
R2 3
1. (3/11) 2. (a) J0A/3; (b) 2J0 A/3 3. R  5 4. (i) 10.52 ; (ii) 0.3125 
1
5. 7/5 times the length of any side of the square

(k  1) 2 k (k  1) / k 2 V0

6. (i) ; (ii) 7. 4/9 kg/sec, 450 sec 8.(i) Vab = –12 V, (ii) 3 amp from b to a
k (k  1) R3

 0L  1 V0 A  e  V ( e  x /L  e 1 ) 22
9. 11  6 2 10. R  1   ; I    ; V 0 11.
1. 7.2 m 13.  Vn
A  e  0L  e  1 1  e 1 9

5
14.  eff   15. 233.3, 144V 16. (a) 6m, (b) 117. (a) 1.01 W, (b) 0-5A, 0-10 V, (c) 0.05 A
4

Exercise-V

V1r2  V2r1 r1 r2
1. 20/3 V 2. (i) D; (ii) M–1 L–3 T3 A2; (iii) , 3. D 4. A
r1  r2 r1  r2

x
A (y) J (1-y) B C
D
5. A 6. D G

7. (a) No, (b) (c) 8 8. A 9. A


10. Battery should be connected across A and B. Out put can be taken across the terminals A and C or B and C
Voltmeter

G1

Ammeter

11. A 12. C 13. G2


14. B 15. D

16. This is true for r1 = r2; So R2 given most accurate value 17. A 18. A 19. C
20. D 21. A,D 22. C 23. D 24. C 25. 4
26. B 27. 5 28 A, B, C, D

394,50 - Rajeev Gandhi Nagar Kota, Ph. No. : 93141-87482, 0744-2209671


IVRS No : 0744-2439051, 52, 53, www. motioniitjee.com , info@motioniitjee.com
Downloaded from www.iitjeephysics4u.com

You might also like